8 класс

Решебник по геометрии 8 класс атанасян бутузов кадомцев позняк: Решебник и ГДЗ по геометрии 7

Содержание

Решебник и ГДЗ по геометрии 7

Авторы: Л.С. Атанасян, В.Ф. Бутузов, С.Б. Кадомцев, Э.Г. Позняк, И.И. Юдин.

В 7, 8, 9 классах обычных общеобразовательных школ, профильных лицеев, гимназий с углубленным изучением отдельных предметов школьники не всегда могут самостоятельно справиться с задачами по геометрии (рабочая программа: Л.С. Атанасян). Именно поэтому многие пользуются решебником с верными ответами, который размещен в интернете. Они находят в ГДЗ по геометрии (авторы: Л.С. Атанасян, В.Ф. Бутузов, С.Б. Кадомцев, Э.Г. Позняк, И.И. Юдин) подробное решение заданий по номерам упражнений, а дальше действуют разными способами (в зависимости от успеваемости и уровня сложности):

  • переписывают в тетрадь;
  • разбирают материал на приведенных примерах;
  • пытаются воспроизвести по памяти или решить по аналогии;
  • изучают возможные варианты построения дополнительных чертежей;
  • готовятся к контрольным работам, экзаменам, олимпиадам, геометрическим тестам.

Правильные ответы помогут ученикам основательно подготовиться к уроку по уже изученной или новой теме. При наличии пропусков старшеклассники могут разобрать тему без учителя, посмотрев, как одноклассники решали в классе, или изучить оформление онлайн в проверочных работах – на сайте они всегда соответствуют нормам ФГОС.

Результаты использования ГДЗ по геометрии за 7 — 9 класс от Атанасяна

Учителя заметили положительную тенденцию от появления онлайн-решебников Атанасяна за 7-9 класс, которые общедоступны всем. Зная, что ученики заглядывают в них практически ежедневно, педагоги, репетиторы и родители не ограничивают их в этом, поскольку на занятиях ощущается практическая польза от этого ресурса. Дети стали значительно лучше учиться, чаще тянут руку. Появилась уверенность в себе, обучающиеся не боятся отвечать устно или письменно, смело выходят к доске и быстро решают любые заданные им задачи.

Повысилась общая успеваемость, что естественным образом сказывается на сдаче выпускниками ЕГЭ по математике. Они набирают большее количество баллов, получают реальную возможность поступления в выбранные ими вузы.

Важно отметить, что отстающие учащиеся теперь подтягиваются в знаниях без репетитора. Главное, объяснить им, как разумно пользоваться сайтом, открывать его не только для автоматического списывания, но и вдумчивого разбора номеров.

ГДЗ Геометрия 8 класс Л.С. Атанасян, В.Ф. Бутузов, С.Б. Кадомцев, Э.Г. Позняк (2014) . Ответы и решения

Для поддержки школьников были созданы специальные справочники-помощники, которые называются ГДЗ. Эти книги прошли достаточно длинный путь развития и стали очень популярными в системе образования. Пособия такого формата используют не только ученики, но и взрослые (родители, педагоги). Секрет любви к решебникам, особенно написанным Атанасяном Л.С., кроется в их простой структуре, благодаря которой работать с ними легко и быстро. Кроме того, ГДЗ приносят в жизнь учеников множество положительных тенденций. Давайте рассмотрим некоторые из них.

Польза, которую приносят решебники по геометрии

Такие книги, как ГДЗ по геометрии 8 класс Атанасян были целенаправленно созданы для помощи детям с выполнением заданий различной сложности и усвоением новых тем. Хочется отметить, что они вполне справляются с данной задачей. Но на этом их плюсы не заканчиваются. Решебники играют очень важную роль в повышении успеваемости учеников, которые пользуясь таким видом учебников, получают только высокие баллы за домашние задания.

Кроме того, ученики становятся самостоятельными и уверенными в своих силах. Это происходит благодаря тому, что они могут справляться даже с самыми сложными заданиями без чьей-либо помощи. Все это мотивирует детей добывать новые знания и занимать лидерские позиции в классе.

Еще одним положительным аспектом работы с ГДЗ Атанасяна по геометрии стоит считать то, что эти справочники освобождают много времени. Теперь у школьников появится возможность в полной мере заниматься своими увлечениями и интересами. Если у Вас возникает волнение по поводу того, что Вам не удастся отыскать качественные решебники – откиньте его навсегда. С появлением нашего образовательного сайта VIPGDZ у Вас всегда под рукой будут необходимые процессы выполнений заданий и ответы на них.

Лучшие правильные решения живут на сайте VIPGDZ.ru

Наш портал VIPGDZ сделал все возможное, чтобы в Вашем распоряжении круглосуточно были нужные ответы по геометрии. Хочется отметить, что наш сайт выгодно выделяется среди всех других ресурсов подобного типа по нескольким параметрам: — свободный доступ ко всем материалам на страницах VIPGDZ. Всю информацию, которую мы предлагаем, Вы можете просматривать абсолютно бесплатно; — отсутствие регистрации, что сэкономит Ваше время; — наличие мобильной версии, которая поспособствует тому, чтобы независимо от Вашего места положения, готовые домашние задания всегда были под рукой. Теперь ими можно пользоваться не только с компьютера, но и с любого другого электронного гаджета; — удобный интерфейс нашего портала, которой сделает работу школьников с книгами в режиме онлайн простой и легкой. Благодаря ему найти интересующее ГДЗ удастся всего за несколько секунд.

Для максимально эффективной работы учеников 8 класса с решебниками на нашем портале VIPGDZ, мы советуем добавить страницу с ним в закладки Вашего браузера.

Учитесь с хорошим настроением и получайте высокие баллы по геометрии вместе с нами!

ГДЗ по Геометрии 7-9 класс

ГДЗ по геометрии за 7-9 класс Атанасян – это сборник готовых решений по всем заданиям одноименного учебного пособия, составленного коллективом российских авторов – Л.С. Атанасяном, В. Ф. Бутузовым, С. Б. Кадомцевым и др. Решебник ориентирован на помощь родителям, желающим помочь своим детям в выполнении домашних заданий; а также на школьников, стремящихся самостоятельно разобраться в ходе решения геометрических задач.

Решебник по геометрии за 7-9 класс от Атанасян Л.С. – основа качественных домашних работ

Упражнения практикума по геометрии не всегда понятны для родителей, помогающих своим детям выполнять домашние задания. Школьники же порой не успевают понять алгоритма выполнения упражнений ввиду большой нагрузки в средней и старшей школе.

И тем, и другим может помочь решебник по геометрии за 7-9 класс Атанасяна, в котором представлены пошаговые алгоритмы выполнения заданий и готовые ответы.

Особенно удобно пользоваться пособием в рамках нашего сайта, преимуществами которого выступают:

  • Регулярное обновление онлайн-решений для их полного соответствия требованиям школьной программы;
  • Удобный поиск позволяет ввести в поисковую строку номер упражнения или часть задания – сделать это можно с любой страницы сайта;
  • Возможность просмотра информации с компьютера, планшета или смартфона.

Такие механизмы способствуют экономии времени. К тому же, пошаговые алгоритмы решения позволяют избежать найма дорогостоящих репетиторов и самостоятельно разобраться в сложных геометрических задачках.

Гдз по геометрии за 7-9 класс Атанасян, Бутузов, Кадомцев – базовый курс с задачами 

Решебник, представленный на сайте, это сборник выполненных заданий, по учебнику по алгебре Атанасяна Л.С., выпущенному в 2014 году издательством «Просвещение». Пособие включает в себя 131 тему, которые разделены на 4 главы.

Учебник знакомит школьников с такими базовыми понятиями геометрии, как:

  • Луч, прямая, отрезок и угол и особенностями их измерения;
  • Треугольники, их свойства, виды, законы равенства и подобия;
  • Параллельность и перпендикулярность прямых и вытекающие из них свойства;
  • Виды многоугольников и их ключевые свойства;
  • Окружность и расчет ее длины, круг и его площадь;
  • Виды векторов и математические действия с ними.

Отдельную часть учебника представляют задачи повышенной сложности и примеры на повторение материала, изученного в предыдущих классах.

Поскольку сборник готовых домашних заданий включает в себя не только онлайн-ответы на задачи и примеры курса геометрии за 7-9 класс, но и детальный алгоритм решения. Такая структура помогает не только в выполнении домашних заданий, но и содействует подготовке к ГИА и ЕГЭ.

Геометрия 7 8 9 класс Атанасян

Геометрия 7 8 9 класс Атанасян

СПИСОК НОМЕРОВ ЗАДАЧ:

       1  2  3  4  5  6  7  8  9 

 10  11  12  13  14  15  16  17  18  19 

 20  21  22  23  24  25  26  27  28  29 

 30  31  32  33  34  35  36  37  38  39 

 40  41  42  43  44  45  46  47  48  49 

 50  51  52  53  54  55  56  57  58  59 

 60  61  62  63  64  65  66  67  68  69 

 70  71  72  73  74  75  76  77  78  79 

 80  81  82  83  84  85  86  87  88  89 

 90  91  92  93  94  95  96  97  98  99 

100

 100  101  102  103  104  105  106  107  108  109 

 110  111  112  113  114  115  116  117  118  119 

 120  121  122  123  124  125  126  127  128  129 

 130  131  132  133  134  135  136  137  138  139 

 140  141  142  143  144  145  146  147  148  149 

 150  151  152  153  154  155  156  157  158  159 

 160  161  162  163  164  165  166  167  168  169 

 170  171  172  173  174  175  176  177  178  179 

 180  181  182  183  184  185  186  187  188  189 

 190  191  192  193  194  195  196  197  198  199 

200

 200  201  202  203  204  205  206  207  208  209 

 210  211  212  213  214  215  216  217  218  219 

 220  221  222  223  224  225  226  227  228  229 

 230  231  232  233  234  235  236  237  238  239 

 240  241  242  243  244  245  246  247  248  249 

 250  251  252  253  254  255  256  257  258  259 

 260  261  262  263  264  265  266  267  268  269 

 270  271  272  273  274  275  276  277  278  279 

 280  281  282  283  284  285  286  287  288  289 

 290  291  292  293  294  295  296  297  298  299 

300

 300  301  302  303  304  305  306  307  308  309 

 310  311  312  313  314  315  316  317  318  319 

 320  321  322  323  324  325  326  327  328  329 

 330  331  332  333  334  335  336  337  338  339 

 340  341  342  343  344  345  346  347  348  349 

 350  351  352  353  354  355  356  357  358  359 

 360  361  362  363  364  365  366  367  368  369 

 370  371  372  373  374  375  376  377  378  379 

 380  381  382  383  384  385  386  387  388  389 

 390  391  392  393  394  395  396  397  398  399 

400

 400  401  402  403  404  405  406  407  408  409 

 410  411  412  413  414  415  416  417  418  419 

 420  421  422  423  424  425  426  427  428  429 

 430  431  432  433  434  435  436  437  438  439 

 440  441  442  443  444  445  446  447  448  449 

 450  451  452  453  454  455  456  457  458  459 

 460  461  462  463  464  465  466  467  468  469 

 470  471  472  473  474  475  476  477  478  479 

 480  481  482  483  484  485  486  487  488  489 

 490  491  492  493  494  495  496  497  498  499 

500

 500  501  502  503  504  505  506  507  508  509 

 510  511  512  513  514  515  516  517  518  519 

 520  521  522  523  524  525  526  527  528  529 

 530  531  532  533  534  535  536  537  538  539 

 540  541  542  543  544  545  546  547  548  549 

 550  551  552  553  554  555  556  557  558  559 

 560  561  562  563  564  565  566  567  568  569 

 570  571  572  573  574  575  576  577  578  579 

 580  581  582  583  584  585  586  587  588  589 

 590  591  592  593  594  595  596  597  598  599 

600

 600  601  602  603  604  605  606  607  608  609 

 610  611  612  613  614  615  616  617  618  619 

 620  621  622  623  624  625  626  627  628  629 

 630  631  632  633  634  635  636  637  638  639 

 640  641  642  643  644  645  646  647  648  649 

 650  651  652  653  654  655  656  657  658  659 

 660  661  662  663  664  665  666  667  668  669 

 670  671  672  673  674  675  676  677  678  679 

 680  681  682  683  684  685  686  687  688  689 

 690  691  692  693  694  695  696  697  698  699 

700

 700  701  702  703  704  705  706  707  708  709 

 710  711  712  713  714  715  716  717  718  719 

 720  721  722  723  724  725  726  727  728  729 

 730  731  732  733  734  735  736  737  738  739 

 740  741  742  743  744  745  746  747  748  749 

 750  751  752  753  754  755  756  757  758  759 

 760  761  762  763  764  765  766  767  768  769 

 770  771  772  773  774  775  776  777  778  779 

 780  781  782  783  784  785  786  787  788  789 

 790  791  792  793  794  795  796  797  798  799 

800

 800  801  802  803  804  805  806  807  808  809 

 810  811  812  813  814  815  816  817  818  819 

 820  821  822  823  824  825  826  827  828  829 

 830  831  832  833  834  835  836  837  838  839 

 840  841  842  843  844  845  846  847  848  849 

 850  851  852  853  854  855  856  857  858  859 

 860  861  862  863  864  865  866  867  868  869 

 870  871  872  873  874  875  876  877  878  879 

 880  881  882  883  884  885  886  887  888  889 

 890  891  892  893  894  895  896  897  898  899 

900

 900  901  902  903  904  905  906  907  908  909 

 910  911  912  913  914  915  916  917  918  919 

 920  921  922  923  924  925  926  927  928  929 

 930  931  932  933  934  935  936  937  938  939 

 940  941  942  943  944  945  946  947  948  949 

 950  951  952  953  954  955  956  957  958  959 

 960  961  962  963  964  965  966  967  968  969 

 970  971  972  973  974  975  976  977  978  979 

 980  981  982  983  984  985  986  987  988  989 

 990  991  992  993  994  995  996  997  998  999 

1000

 1000  1001  1002  1003  1004  1005  1006  1007  1008  1009 

 1010  1011  1012  1013  1014  1015  1016  1017  1018  1019 

 1020  1021  1022  1023  1024  1025  1026  1027  1028  1029 

 1030  1031  1032  1033  1034  1035  1036  1037  1038  1039 

 1040  1041  1042  1043  1044  1045  1046  1047  1048  1049 

 1050  1051  1052  1053  1054  1055  1056  1057  1058  1059 

 1060  1061  1062  1063  1064  1065  1066  1067  1068  1069 

 1070  1071  1072  1073  1074  1075  1076  1077  1078  1079 

 1080  1081  1082  1083  1084  1085  1086  1087  1088  1089 

 1090  1091  1092  1093  1094  1095  1096  1097  1098  1099 

1100

 1100  1101  1102  1103  1104  1105  1106  1107  1108  1109 

 1110  1111  1112  1113  1114  1115  1116  1117  1118  1119 

 1120  1121  1122  1123  1124  1125  1126  1127  1128  1129 

 1130  1131  1132  1133  1134  1135  1136  1137  1138  1139 

 1140  1141  1142  1143  1144  1145  1146  1147  1148  1149 

 1150  1151  1152  1153  1154  1155  1156  1157  1158  1159 

 1160  1161  1162  1163  1164  1165  1166  1167  1168  1169 

 1170  1171  1172  1173  1174  1175  1176  1177  1178  1179 

 1180  1181  1182  1183  1184  1185  1186  1187  1188  1189 

 1190  1191  1192  1193  1194  1195  1196  1197  1198  1199 

1200

 1200  1201  1202  1203  1204  1205  1206  1207  1208  1209 

 1210  1211  1212  1213  1214  1215  1216  1217  1218  1219 

 1220  1221  1222  1223  1224  1225  1226  1227  1228  1229 

 1230  1231  1232  1233  1234  1235  1236  1237  1238  1239 

 1240  1241  1242  1243  1244  1245  1246  1247  1248  1249 

 1250  1251  1252  1253  1254  1255  1256  1257  1258  1259 

 1260  1261  1262  1263  1264  1265  1266  1267  1268  1269 

 1270  1271  1272  1273  1274  1275  1276  1277  1278  1279 

 1280  1281  1282  1283  1284  1285  1286  1287  1288  1289 

 1290  1291  1292  1293  1294  1295  1296  1297  1298  1299 

1300

 1300  1301  1302  1303  1304  1305  1306  1307  1308  1309 

 1310 

Контрольные вопросы к главе VI. Контрольные вопросы к главе VI «Проблемы строительства»

Готовое домашнее задание к учебнику геометрии для учащихся 7-9 классов, авторы: Л.С. Атанасян, В.Ф. Бутузов, С. Кадомцев, Э. Позняк, И. Юдина, Издательство «Образование» на 2015-2016 учебный год.

Ребята, в 7-9 классах вы будете изучать такой интересный предмет, как геометрия. Чтобы избежать дальнейших проблем с пониманием этого урока, вам нужно усердно работать с самого начала.

На предыдущих занятиях вы уже встречали некоторые геометрические фигуры … В этом шумихе вы расширите этот минимум знаний. Весь курс разделен на два раздела: планиметрия и стереометрия. В 7 и 8 классах вы будете рассматривать фигуры на плоскости — это разрез планиметрии. В 9 классе свойства фигур в пространстве — стереометрия.

Часто возникает ситуация, когда невозможно по условию сделать правильный рисунок, прорисовать все детали в пространстве, и тогда геометрия кажется вам невыносимым объектом.Если у вас начинаются такие трудности, то рекомендуем использовать нашу ГДЗ по геометрии на 7-9 л. Атанасяна, который размещен ниже.

ГДЗ Геометрия 7 класс Рабочую тетрадь Атанасяна можно скачать.

ГДЗ Геометрия 8 класс Рабочую тетрадь Атанасяна можно скачать.

ГДЗ Геометрия 9 класс Рабочую тетрадь Атанасяна можно скачать.

ГДЗ к дидактическим материалам по геометрии за 7 класс Зив Б.Г. можно скачать.

ГДЗ к дидактическим материалам по геометрии для 8 класса Зив Б.Г. можно скачать.

ГДЗ к дидактическим материалам по геометрии для 9 класса Зив Б.Г. можно скачать.

ГДЗ к самостоятельным и контрольным работам по геометрии для 7-9 классов Иченская М.А. можно скачать.

ГДЗ к сборнику заданий по геометрии для 7 класса Ершова А.П. можно скачать.

ГДЗ к сборнику заданий по геометрии для 8 класса Ершова А.П. можно скачать.

ГДЗ к рабочей тетради по геометрии для 9 класса Мищенко Т.М. можно скачать.

ГДЗ для тематических зачетов по геометрии для 7 класса Мищенко Т.М. можно скачать.

ГДЗ для тематических зачетов по геометрии для 8 класса Мищенко Т.М. можно скачать

Современные дети регулярно сталкиваются с ситуациями, когда с домашним заданием возникают определенные проблемы … Причины таких обстоятельств могут быть самые разные — лень, болезнь, невнимательность. Особенно это касается геометрии, в которой много непонятных упражнений.Если есть проблемы, то старшеклассники — начинают лихорадочно искать варианты решения подобных трудностей. Действительно, кто-то обращается к родственникам, друзьям, наставникам, а кто-то ищет GDZ , которые сделаны профессионалами, которые не ошибаются.

Благодаря стремительному развитию Интернет-технологий появилась прекрасная возможность находить нужные задачи с помощью специализированной платформы. Главное — ответственно отнестись к вопросу, чтобы готовые Д / З были качественными и полностью понятными.Конечно, нужно доверять данным, размещенным на тех интернет-ресурсах, которые успели зарекомендовать себя с сильной стороны. Только такие ресурсы содержат качественную информацию о домашних заданиях, которую вы можете использовать при необходимости.

Представленное решение будет рациональным выбором для определенных ситуаций. Он содержит наиболее грамотные и развернутые ответы, по геометрии для учащихся 7-9 классов … Они подходят для учебников авторов — Атанасяна и Бутузова … Вы можете быстро сравнить результаты на этом сайте и повысить реальный уровень знаний и эрудиции в столь сложной теме. Поэтому его часто используют школьники и их родители.

Высококвалифицированная администрация портала позаботилась о том, чтобы материал был написан в доступной и понятной форме. Если издаются новые книги, то сразу появляются ответы на новые вопросы. Многочисленные посетители портала уже неоднократно могли убедиться в этом.

Важно понимать, что если возникают трудности с изучением базовых дисциплин, то следует убедиться, что они разрешены. Подтягивать не нужно, это приводит к крайне неприятным последствиям … Эта онлайн-страница может послужить отличным местом, где можно проверить правильность исполнения цифр, которые поставили учителя. Многие подростки уже используют его и оставили много хороших отзывов о нем. Это неудивительно, благодаря ей есть прекрасная возможность получить высокие оценки и добиться лучшей успеваемости в школе.

ГДЗ Геометрия 7 класс Рабочую тетрадь Атанасяна можно скачать.

ГДЗ Геометрия 8 класс Рабочую тетрадь Атанасяна можно скачать.

ГДЗ Геометрия 9 класс Рабочую тетрадь Атанасяна можно скачать.

ГДЗ к дидактическим материалам по геометрии за 7 класс Зив Б.Г. можно скачать.

ГДЗ к дидактическим материалам по геометрии для 8 класса Зив Б.Г. можно скачать.

ГДЗ к дидактическим материалам по геометрии для 9 класса Зив Б.Г. можно скачать.

ГДЗ для самостоятельных и контрольных работ по геометрии для 7-9 классов Иченская М.А. можно скачать.

ГДЗ к сборнику заданий по геометрии для 7 класса Ершова А.П. можно скачать.

ГДЗ к сборнику заданий по геометрии для 8 класса Ершова А.П. можно скачать.

ГДЗ к рабочей тетради по геометрии для 9 класса Мищенко Т.М. можно скачать

1. Что называется соотношением двух отрезков?

2. В каком случае говорится, что отрезки AB и CD пропорциональны отрезкам A 1 B 1 и C 1 D 1?

3.Дайте определение подобным треугольникам.

4. Сформулируйте и докажите теорему о соотношении площадей подобных треугольников.

5. Сформулируйте и докажите теорему, выражающую первый критерий подобия треугольников.

6. Сформулируйте и докажите теорему, выражающую второй критерий подобия треугольников.

7. Сформулируйте и докажите теорему, выражающую третий критерий подобия треугольников.

8. Какой отрезок называется средней линией треугольника? Сформулируйте и докажите теорему о средней линии треугольника.

9. Докажите, что медианы треугольника пересекаются в одной точке, которая делит каждую медиану в соотношении 2: 1, считая от вершины.

10. Сформулируйте и докажите утверждение, что высота прямоугольного треугольника, начерченного из прямого угла вершины, делит треугольник на аналогичные треугольники.

11. Сформулируйте и докажите утверждение о пропорциональных отрезках в прямоугольном треугольнике.

12. Приведите пример решения строительной задачи методом подобия.

13. Расскажите, как определить высоту объекта и расстояние до недоступной точки на земле.

14. Объясните, какие две фигуры называются похожими. Каков коэффициент подобия форм?

15. Что называют синусом, косинусом, тангенсом острого угла прямоугольного треугольника?

16. Докажите, что если острый угол одного прямоугольного треугольника равен острому углу другого прямоугольного треугольника, то синусы этих углов равны, косинусы этих углов равны и касательные этих углов равны.

17. Какое равенство называется основным тригонометрическим тождеством?

18. Какие значения синуса, косинуса и тангенса для углов 30 °, 45 °, 60 °? Обоснуйте ответ.

Дополнительные задачи

604. Треугольники ABC и A 1 B 1 C 1 подобны, AB = 6 см, BC — 9 см, C A = 10 см. Наибольшая сторона треугольника A 1 B 1 C 1 равна 7,5 см. Найдите две другие стороны треугольника A 1 IN 1 WITH 1.

605. Диагональ AC трапеции ABCD делит ее на два подобных треугольника.Докажите, что AC 2 = a b, где a и b — основания трапеции.

606. Биссектрисы MD и NK треугольника MNP пересекаются в точке O. Найдите соотношение ОК: ВКЛ, если MN = 5 см, NP = 3 см, MP = 7 см.

607. Основание равнобедренного треугольника относится к боковой стороне как 4: 3, а высота, обращенная к основанию, составляет 30 см. Найдите отрезки, на которые эта высота делится биссектрисой угла при основании.

608. На продолжении боковой стороны OB равнобедренного треугольника AO B с основанием AB берется точка C так, чтобы точка B находилась между точками O и C.Отрезок AC пересекает биссектрису угла AOB в точке M. Докажите, что AM

609. На стороне BC треугольника ABC взята точка D, так что Докажите, что AD является биссектрисой треугольника ABC.

610. Прямая, параллельная стороне AB треугольника ABC, делит сторону AC в соотношении 2: 7, считая от вершины A. Найдите стороны отсеченного треугольника, если AB = 10 см. , BC = 18 см, CA = 21,6 см.

611. Докажите, что медиана AM треугольника ABC делит пополам любой отрезок, параллельный стороне BC, концы которого лежат на сторонах AB и AC.

612. Две опоры AB и CD разной длины a и b устанавливаются вертикально на определенном расстоянии друг от друга, как показано на рисунке 210. Концы A и D, B и C соединены веревками, пересекающимися в точке O 2. Докажите, что в соответствии с рисунком:

Найдите x и докажите, что x не зависит от расстояния d между полюсами AB и CD.

Рис. 210

613. Докажите, что треугольники ABC и A 1 B 1 C 1 подобны, если:

и), где VM и B 1 M 1 — медианы треугольников;

б) ∠A = ∠A 1, где BH и B 1 H 1 — высоты треугольников ABC и A 1 B 1 C 1.

614. Диагонали прямоугольной трапеции ABCD с прямым углом A взаимно перпендикулярны. Основание AB — 6 см, сторона AD — 4 см. Найдите DC, DB и CB.

615. * Отрезок с концами на боковых сторонах трапеции параллелен ее основаниям и проходит через точку пересечения диагоналей. Найдите длину этого отрезка, если основания трапеции равны a и b.

616. Докажите, что вершины треугольника равноудалены от линии, содержащей его среднюю линию.

617. Докажите, что середины сторон ромба являются вершинами прямоугольника.

618. Точки M и N являются серединами сторон CD и BC параллелограмма ABCD соответственно. Докажите, что прямые AM и AN делят диагональ BD на три равные части.

619. Биссектриса внешнего угла в вершине A треугольника ABC пересекает прямую BC в точке D. Докажите, что.

620. В треугольнике ABC (AB ≠ AC) через середину стороны BC проводится прямая, параллельная биссектрисе угла A, которая пересекает прямые AB и AC соответственно в точках D и E.Докажите, что BD = CE.

621. В трапеции ABCD с основаниями AD и BC сумма оснований равна b, диагональ AC равна a, ∠ACB = α. Найдите площадь трапеции.

622. На стороне AD параллелограмма ABCD отмечается точка K так, чтобы AK = 1/4 KD. Диагональ AC и отрезок BK пересекаются в точке P. Найти площадь параллелограмма ABCD, если площадь треугольника ARC равна 1 см 2.

623. В прямоугольной трапеции ABCD с основаниями AD и BC ∠ A = ∠B = 90 °, ∠ACD = 90 °, BC = 4 см, AD = 16 см.Найдите углы C и D трапеции.

624. Докажите, что медианы треугольника разбивают его на шесть треугольников, площади которых попарно равны.

625. AD Основание равнобедренной трапеции ABCD в 5 раз больше основания BC. Высота ЧД пересекает диагональ АС в точке М, площадь треугольника АМГ равна 4 см 2. Найдите площадь трапеции ABCD.

626. Докажите, что треугольники ABC и A 1 B 1 C 1 подобны, если где AD и A 1 D 1 — биссектрисы треугольников.

Строительные задания

627. Дан треугольник ABC. Постройте треугольник A1B1C1, аналогичный треугольнику ABC, площадь которого вдвое больше площади треугольника ABC.

628. Даны три сегмента, длина которых равна соответственно a, b и c. Постройте отрезок прямой, длина которого равна.

629. Постройте треугольник, если даны середины его сторон.

630. Постройте треугольник вдоль стороны и проведите медианы к двум другим сторонам.

Ответы на проблемы

1. Какова последовательность звеньев в цепочке формирования затрат на качество и стоимость фарфоровой посуды?

2. Какие подразделения предприятия обеспечивают качество выпускаемой продукции?

3. Объясните роль отдела планирования, бухгалтерии, отдела подготовки производства в обеспечении качества продукции.

4. Сравните функции отдела закупок и отдела продаж в обеспечении качества продукции.

5. Какие затраты на качество формируются на «исполнительном» уровне отделов?

6. Перечислите состав затрат на управление качеством. Чем они отличаются от серийных?

7. Какие затраты на качество относятся к основным, а какие к дополнительным? Есть ли среди них повторяющиеся?

8. Объясните разницу между внутренней и внешней информацией о качестве продукции.

9. Как ускорить получение выводов о предмете исследования на первичных данных?

10.Каковы формы регистрации данных, позволяющие увидеть взаимосвязь между затратами и факторами, влияющими на них.

11. В чем преимущество сметы расходов по сравнению с другими средствами массовой информации?

12. Перечислите этапы построения диаграммы рассеяния. Можно ли с его помощью определить наличие и направление взаимосвязи между эффективными и факторными показателями?

13. Какое расположение точек на диаграмме рассеяния указывает на положительную, отрицательную корреляцию, ее отсутствие?

14.Каковы принципы использования FSA?

15. Каковы причины классификации функций продукта. Какая между ними связь?

16. Опишите этапы АФН?

17. Что такое принцип Эйзенхауэра в FSA?

18. Можете ли вы использовать табличную форму для определения функций продукта, которые необходимо улучшить или исключить?

19. Что представляет собой матричная таблица выбора продукции для производства? Какие показатели позволяют сделать этот выбор.

20. Как рассчитывается коэффициент корреляции между показателями качества и стоимостью его создания?

21. Как использовать индексный метод для определения влияния качества на себестоимость продукции?

22. Каковы недостатки методов подсчета баллов и цены за единицу? Какова сфера их применения?

23. Где и как применяется показатель доходности?

24. Как рассчитывается обобщающий коэффициент качества?

25. Как определить объем продукции, потерянной предприятием из-за возникновения дефектов, и стоимость их устранения?

26.Каковы направления определения экономической эффективности от внедрения более качественной продукции? Чем они отличаются и что общего при расчете показателя экономической эффективности во всех случаях?

27. В каких областях анализа проектов используются в основном формальные или неформальные методы? Почему?

28. Каковы цели коммерческого анализа?

29. Какие показатели можно использовать для оценки конкурентоспособности продукции?

30. Показать важность анализа проекта и внедрения новой продукции для региона, в котором находится производитель.

31. Отражены ли затраты, связанные с качеством продукта, в цене пункта, в цене за единицу продукта?

32. Отражается ли стоимость качества в показателе рентабельности продукции? Объясните свое мнение.

Контрольные вопросы к главе 8. Контрольные вопросы к главе I. Дополнительные задачи к главе I

Современные дети регулярно сталкиваются с ситуациями, когда возникают определенные проблемы с выполнением домашних заданий. Причины таких обстоятельств могут быть самые разные — лень, болезнь, невнимательность.Особенно это касается геометрии, в которой много непонятных упражнений. Если возникают проблемы, то старшеклассники начинают лихорадочно искать варианты решения таких трудностей. Действительно, кто-то обращается к родственникам, друзьям, наставникам, а кто-то ищет GDZ , которые сделаны профессионалами без ошибок.

Благодаря стремительному развитию Интернет-технологий появилась прекрасная возможность находить нужные задачи с помощью специализированной платформы.Главное — ответственно отнестись к вопросу, чтобы готовые Д / З были качественными и полностью понятными. Конечно, нужно доверять данным, размещенным на тех интернет-ресурсах, которые успели зарекомендовать себя с сильной стороны. Только такие ресурсы содержат качественную информацию о домашних заданиях, которую вы можете использовать при необходимости.

Представленное решение станет рациональным выбором для индивидуальных ситуаций. В нем собраны наиболее грамотные и развернутые ответы по геометрии для учащихся с 7 по 9 классы … Они подходят для учебников авторов — Атанасяна и Бутузова … На этом сайте вы можете быстро сравнить результаты и поднять реальный уровень знаний и эрудиции в столь сложном предмете. Поэтому его часто используют школьники и их родители.

Высококвалифицированная администрация портала позаботилась о том, чтобы материал был написан в доступной и понятной форме. Если издаются новые книги, то сразу появляются ответы на новые вопросы.Многочисленные посетители портала неоднократно могли убедиться в этом.

Важно понимать, что если возникают трудности с изучением базовых дисциплин, то следует убедиться, что они разрешены. Медлить не нужно, это приводит к крайне неприятным последствиям. Эта онлайн-страница может послужить отличным местом, где вы сможете убедиться в правильности выполнения цифр, которые были выставлены учителями. Многие подростки уже пользуются им и оставили много хороших отзывов о нем.Это неудивительно, благодаря ей есть прекрасная возможность получить высокие оценки и добиться лучшей успеваемости в школе.

ГДЗ Геометрия 7 класс Рабочую тетрадь Атанасяна можно скачать.

ГДЗ Геометрия 8 класс Рабочую тетрадь Атанасяна можно скачать.

ГДЗ Геометрия 9 класс Рабочую тетрадь Атанасяна можно скачать.

ГДЗ к дидактическим материалам по геометрии за 7 класс Зив Б.Г. можно скачать.

ГДЗ к дидактическим материалам по геометрии для 8 класса Зив Б.Г. можно скачать.

ГДЗ для дидактических материалов по геометрии для 9 класса Зив Б.Г. можно скачать.

ГДЗ к самостоятельным и контрольным работам по геометрии для 7-9 классов Иченская М.А. можно скачать.

ГДЗ к сборнику заданий по геометрии для 7 класса А.П. Ершова можно скачать.

ГДЗ к сборнику заданий по геометрии для 8 класса Ершова А.П. можно скачать.

ГДЗ к рабочей тетради по геометрии для 9 класса Мищенко Т.М. можно скачать

1. Какова последовательность звеньев в цепочке формирования затрат на качество и стоимость фарфоровой посуды?

2. Какие подразделения предприятия обеспечивают качество выпускаемой продукции?

3. Объясните роль отдела планирования, бухгалтерии, отдела подготовки производства в обеспечении качества продукции.

4. Сравните функции отдела закупок и отдела продаж в обеспечении качества продукции.

5. Какие затраты на качество возникают на «исполнительном» уровне отделов?

6. Перечислите состав затрат на управление качеством. Чем они отличаются от серийных?

7. Какие затраты на качество относятся к основным, а какие к дополнительным? Есть ли среди них повторяющиеся?

8. Объясните разницу между внутренней и внешней информацией о продукте.

9. Как ускорить получение выводов о предмете исследования на первичных данных?

10.Каковы формы регистрации данных, позволяющие увидеть взаимосвязь между затратами и факторами, влияющими на них.

11. В чем преимущество сметы перед другими носителями?

12. Перечислите этапы построения диаграммы рассеяния. Можно ли по нему определить наличие и направление взаимосвязи между эффективными и факторными показателями?

13. Какое расположение точек на диаграмме рассеяния указывает на положительную, отрицательную корреляцию, ее отсутствие?

14.Каковы принципы использования FSA?

15. Каковы причины классификации функций продукта. Какая между ними связь?

16. Опишите этапы АФН?

17. Что такое принцип Эйзенхауэра в FSA?

18. Можете ли вы использовать табличную форму для определения функций продукта, которые необходимо улучшить или исключить?

19. Что представляет собой матричная таблица выбора продукции для производства? Какие показатели позволяют сделать этот выбор.

20. Как рассчитывается коэффициент корреляции между показателями качества и стоимостью его создания?

21. Как использовать индексный метод для определения влияния качества на себестоимость продукции?

22. Каковы недостатки методов подсчета баллов и цены за единицу? Каков их объем?

23. Где и как применяется показатель доходности?

24. Как рассчитывается обобщающий коэффициент качества?

25. Как определить объем продукции, потерянной предприятием из-за возникновения дефектов, и стоимость их устранения?

26.Каковы направления определения экономической эффективности от внедрения более качественной продукции? Чем они отличаются и что общего при расчете показателя экономической эффективности во всех случаях?

27. В каких областях анализа проектов чаще всего используются формальные или неформальные методы? Почему?

28. Каковы цели коммерческого анализа?

29. Какие показатели можно использовать для оценки конкурентоспособности продукции?

30. Показать важность анализа проекта и внедрения новой продукции для региона, в котором находится производитель.

31. Отражены ли затраты, связанные с качеством продукта, в цене пункта, в цене за единицу продукта?

32. Отражается ли стоимость качества в показателе прибыльной продукции? Объясните свое мнение.

Готовое домашнее задание к учебнику геометрии для учащихся 7-9 классов, авторы: Л.С. Атанасян, В.Ф. Бутузов, С. Кадомцев, Э. Позняк, И. Юдин, Издательство «Образование» на 2015-2016 учебный год.

Ребята, в 7-9 классах вы будете изучать такой интересный предмет, как геометрия.Чтобы избежать дальнейших проблем с пониманием этого урока, вам нужно усердно работать с самого начала.

На предыдущих занятиях вы уже встречались с некоторыми геометрическими фигурами … В этом шумихе вы расширите этот минимум знаний. Весь курс разделен на два раздела: планиметрия и стереометрия. В 7 и 8 классах вы будете рассматривать фигуры на плоскости — это разрез планиметрии. В 9 классе свойства фигур в пространстве — стереометрия.

Часто возникает ситуация, когда невозможно по условию сделать правильный рисунок, прорисовать все детали в пространстве, и тогда геометрия кажется вам невыносимым объектом.Если у вас начинаются такие трудности, то рекомендуем использовать нашу ГДЗ по геометрии для 7-9 класса Л.С. Атанасяна, который размещен ниже.

ГДЗ Геометрия 7 класс Рабочую тетрадь Атанасяна можно скачать.

ГДЗ Геометрия 8 класс Рабочую тетрадь Атанасяна можно скачать.

ГДЗ Геометрия 9 класс Рабочую тетрадь Атанасяна можно скачать.

ГДЗ к дидактическим материалам по геометрии за 7 класс Зив Б.Г. можно скачать.

ГДЗ к дидактическим материалам по геометрии для 8 класса Зив Б.Г. можно скачать.

ГДЗ для дидактических материалов по геометрии для 9 класса Зив Б.Г. можно скачать.

ГДЗ для самостоятельных и контрольных работ по геометрии для 7-9 классов Иченская М.А. можно скачать.

ГДЗ к сборнику заданий по геометрии для 7 класса А.П. Ершова можно скачать.

ГДЗ к сборнику заданий по геометрии для 8 класса Ершова А.П. можно скачать.

ГДЗ к рабочей тетради по геометрии для 9 класса Мищенко Т.М. можно скачать.

ГДЗ для тематических зачетов по геометрии для 7 класса Мищенко Т.М. можно скачать.

ГДЗ для тематических зачетов по геометрии для 8 класса Мищенко Т.М. можно скачать

1 Приведите примеры векторных величин, известных вам из курса физики.

2 Дайте определение вектора. Объясните, какой вектор называется нулевым.

3 Какова длина ненулевого вектора? Какова длина нулевого вектора?

4 Какие векторы называются коллинеарными? Нарисуйте сонаправленные векторы и противоположно направленные векторы.

5 Дайте определение равных векторов.

6 Объясните значение выражения: «Вектор отложен из точки A». Докажите, что из любой точки можно отложить вектор, равный заданному, причем только один.

7 Объясните, какой вектор называется суммой двух векторов. Каково правило треугольника для сложения двух векторов?

8 Докажите, что для любого вектора выполняется равенство

9 Сформулируйте и докажите теорему о законах сложения векторов.

10 Какое правило параллелограмма для сложения двух неколлинеарных векторов?

11 Что такое правило многоугольника для добавления нескольких векторов?

12 Какой вектор называется разностью двух векторов? Постройте разницу между двумя заданными векторами.

13 Какой вектор называется противоположным данному? Сформулируйте и докажите теорему о векторной разности.

14 Какой вектор называется произведением этого вектора на заданное число?

15 Что такое произведение

16 Могут ли векторы быть неколлинеарными?

17 Сформулируйте основные свойства умножения вектора на число.

18 Приведите пример применения векторов к решению геометрических задач.

19 Какой сегмент называется средней линией трапеции?

20 Сформулируйте и докажите теорему о средней трапеции.

Дополнительные задачи по главе IX

800. Докажите, что если векторы сонаправлены, то если они противоположны, то

801. Докажите, что для любых векторов выполняются неравенства

802. Точка N отмечена на стороне BC треугольника ABC, так что BN \ u003d 2NC. Выразите вектор через векторы

803. Точки X и Y отмечены на сторонах MN и NP треугольника MNP соответственно, так что

804. Основание AD трапеции ABCD в три раза больше основания BC. .Точка K отмечена на стороне AD так, чтобы выразить векторы через векторы

805. Три точки A, B и C расположены так, что Докажите, что для любой точки O выполняется равенство

806. Точка C делит отрезок AB в отношении m: n, считая от точки A. Докажите, что для любой точки O выполняется равенство

1. Сколько линий вы можете провести через две точки?

2. Сколько общих точек могут иметь две линии?

3. Объясните, что такое линейный сегмент.

4.Объясните, что такое луч. Как обозначаются лучи?

5. Какая форма называется углом? Объясните, что такое верх и боковые стороны угла.

6. Какой угол называется развернутым?

7. Какие цифры называются равными?

8. Объясните, как сравнить две строки.

9. Какая точка называется средней точкой отрезка?

10. Объясните, как сравнивать два угла.

11. Какой луч называется биссектрисой угла?

12.Точка C делит отрезок AB на два отрезка. Как найти длину отрезка AB, если известны длины отрезков AC и CB?

13. Какие инструменты используются для измерения расстояний?

14. В каком градусе измеряется угол?

15. Луч OS делит угол AOB на два угла. Как найти градусную меру угла AOB, если градусная мера углов AOC и COB известна?

16. Какой угол называется острым? непосредственный? глупый?

17.Какие углы называются смежными? Какова сумма смежных углов?

18. Какие углы называются вертикальными? Каким свойством обладают вертикальные углы?

19. Какие прямые называются перпендикулярными?

20. Объясните, почему две прямые, перпендикулярные третьей, не пересекаются.

21. Какие устройства используются для построения прямых углов на земле?

Дополнительные задачи по главе I

71. Отметьте четыре точки так, чтобы никакие три не лежали на одной прямой. Проведите прямую линию через каждую пару точек.Сколько прямых линий получилось?

72. Даны четыре прямые, две из которых пересекаются. Сколько точек пересечения у этих линий, если через каждую точку пересечения проходят только две линии?

73. Сколько неразвернутых углов образуется на пересечении трех прямых, проходящих через одну точку?

74. Точка N лежит на отрезке MP. Расстояние между точками M и P составляет 24 см, а расстояние между точками N и M в два раза больше расстояния между точками N и P.Найдите расстояние:

75. Три точки K, L, M лежат на одной прямой, KL = 6 см, LM = 10 см. Какое может быть расстояние КМ? Для каждого из возможных случаев сделайте чертеж.

76. Отрезок AB длины a разделен точками P и Q на три сегмента AP, PQ и QB так, что AP — 2PQ = 2QB. Найдите расстояние между:

77. Отрезок длиной m делится:

Найдите расстояние между серединами крайних частей.

78. Отрезок длиной 36 см разделен на четыре не равные друг другу части.Расстояние между серединами наконечников 30 см. Найдите расстояние между серединами средних частей.

79. Точки A, B и C лежат на одной прямой, точки M и N являются серединами отрезков AB и AC. Докажите, что BC = 2MN.

80. Известно, что ZAOB = 35 °, ZBOC = 50 °. Найдите угол AOC. Для каждого из возможных случаев нарисуйте рисунок с помощью линейки и транспортира.

81. Угол hk равен 120 °, а угол hm равен 150 °.Найдите угол км. Для каждого из возможных случаев сделайте чертеж.

82. Найдите смежные углы, если:

83. Найдите угол, образованный биссектрисами двух смежных углов.

84. Докажите, что биссектрисы вертикальных углов лежат на одной прямой.

85. Докажите, что если биссектрисы углов ABC и CBD перпендикулярны, то точки A, B и D лежат на одной прямой.

86. Даны две пересекающиеся прямые a и b и точка A, которая не лежит на этих прямых.Прямые m и n проводятся через точку A так, что m⊥a, n⊥b. Докажите, что прямые m и n не совпадают.

Определение параллельной граммы и ее свойств.

В сегодняшнем уроке мы повторяем основные свойства параллелограмма, а затем обращаем внимание на рассмотрение первых двух знаков параллелограмма и их доказательство. Во время доказательства мы вспомним применение знаков равенства треугольников, которое мы изучили в прошлом году и повторили в первом уроке.В конце будет приведен пример применения изученных свойств параллелограмма.

Тема: четырехугольники

Урок: Знаки параллелограмма

Начнем с того, что вспомним определение параллелограмма.

Определение. Параллелограмм — Четырехугольник, две противоположные стороны которого параллельны (см. Рис. 1).

Рис. 1. Поллограмма

Запомните основные свойства параллелограмма :

Для того, чтобы иметь возможность пользоваться всеми этими качествами, необходимо быть уверенным, что фигура об этом и есть речевой параллелограмм.Для этого нужно знать такие факты, как знаки параллелограмма. Первые два из них мы рассмотрим сегодня.

Теорема. Первая особенность параллелограмма. Если в четырехугольнике две противоположные стороны равны и параллельны, то в этом четырехугольнике — параллелограмм . .

Рис. 2. Первая особенность параллелограмма

Доказательства. Проведем в четырехугольнике диагональ (см. Рис. 2), она разбила ее на два треугольника.Пишем, что знаем об этих треугольниках:

по первому признаку равенства треугольников.

Из равенства этих треугольников следует, что на основании параллельности прямых при пересечении их секущих. У нас это:

Доказано.

Теорема. Второй знак параллелограмма. Если в четырехугольнике каждые две противоположные стороны равны, то в этом четырехугольнике — параллелограмм . .

Рис. 3. Параллелограмм второго знака

Доказательства. Проводим в четырехугольнике диагональ (см. Рис. 3), он разбивает его на два треугольника. Пишем, что знаем об этих треугольниках, исходя из формулировки теоремы:

По третьему признаку равенства треугольников.

Из равенства треугольников следует, что на основе параллельности прямых с пересечением их последовательных. Получаем:

параллелограмм по определению.Q.E.D.

Доказано.

Рассмотрим пример нанесения знаков параллелограмма.

Пример 1. В выпуклом четырехугольнике найти: а) углы четырехугольника; б сторона.

Решение. Картинка с рисом. четыре.

Фиг.4

Поллограмм по первому знаку параллелограмма.

Параллелограмм называется четырехугольником, у которого противоположные стороны параллельны, т.е. лежат на параллельной прямой

Свойства Pollogram:
Теорема 22. Противоположные стороны параллельного грамма равны.
Доказательства. В параллелограмме АВД проведем диагональ АК. Треугольники ACD и DCA равны, так как имеют общую сторону переменного тока и две пары равных углов. На него отправлено: ∠ Sav = ∠ ACD, ∠ DC = ∠ DAC (как нижележащие углы при параллельных прямых AD и Sun). Итак, Av = CD и Sun = AD, как соответствующие стороны равных треугольников, гл.Т.Д. Из равенства этих треугольников следует также равенство соответствующих углов треугольников:
Теорема 23. Противоположные углы параллелограмма равны: ∠ A = ∠ C и ∠ B = ∠ D.
Равенство первой пары происходит из равенства треугольников AVD и CBD, а второй — ABC и ACD.
Теорема 24.
Соседние углы параллелограмма, т.е. углы, прилегающие к одной стороне, составляют 180 градусов.
Это потому, что это внутренние односторонние углы.
Теорема 25.
Диагональ параллелограмма должна делиться друг на друга в точке их пересечения пополам.
Доказательства. Рассмотрим треугольники Bos и AOD. Согласно первому свойству AD = Sun ∠ OAD = ∠ ASS и ∠ ODA = ∠ ABS, поскольку лежит в основе под параллельными прямыми AD и Sun. Следовательно, треугольники ВС и АОП равны по бокам и прилегающим к ним углам. Итак, при = OD и AO = OS, как соответствующие стороны равных треугольников, гл.Т.Д.

Знаки параллелограмма
Теорема 26.
Если противоположные стороны четырехугольника попарно равны, то это параллелограмм.
Доказательства. Предположим, что в четырехугольнике AVD есть AD и Sun, AV и CD соответственно (Riga2). Проведем диагональ динамиков. Треугольники ABC и ACD равны трем сторонам. Тогда углы вас и DSA равны и, следовательно, автоматически параллельны CD. Параллельность сторон Солнца и AD следует из равенства углов CAD и DR.
Теорема 27.
Если противоположные углы четырехугольника попарно равны, то это параллелограмм.
Пусть ∠ A = ∠ C и ∠ B = ∠ D.Поскольку ∠ A + ∠ B + ∠ C + ∠ D = 360 O, то ∠ A + ∠ B = 180 O и AD сторона и солнце параллельны (исходя из параллельности прямой). Мы также доказываем параллелизм сторон AV и CD и заключаем, что AVD является параллелограммом по определению.
Теорема 28.
Если прилегающие углы четырехугольника, т.е. углы, прилегающие к одной стороне, составляют 180 градусов, это параллелограмм.
Если внутренние односторонние углы в сумме равны 180 градусам, то прямолинейно быстро.Итак, AV paralo CD и sun paral ad. Четырехугольник по определению оказывается параллелограммом.
Теорема 29.
Если диагональ четырехугольника в точке пересечения пополам делится пополам, то четырехугольник является параллелограммом.
Доказательства. Если AO = OS, IC = OD, то треугольники AOD и VOS равны, так как имеют равные углы (по вертикали) вверху заключенных между парами равных сторон. Из равенства треугольников заключаем, что AD и Sun равны.Также равны стороны AB и CD, и четырехугольник оказывается параллелограммом на основании 1.
Теорема 30.
Если у четырехугольника есть пара равных параллельных сторон, то это параллелограмм.
Предположим, что у квадрилатера AVD стороны AB и CD параллельны и равны. Проводим диагонали АК и КД. Исходя из параллельности этих прямых, равенство должно быть равным нижележащим углам AVO = CDO и VAO = OSD. Треугольники AVO и CDO равны по бокам и примыкают к его углам.Следовательно, АО = ОС, I = OD, т.е. диагональ точки пересечения делится пополам и четырехугольник получается параллелограммом на основании 4.

В геометрии бывают частные случаи параллелограмм.

Чтобы определить, является ли эта фигура параллелограммом, существует ряд особенностей. Рассмотрим три основных особенности параллелограмма.

1 знак параллелограмма

Если в четырехугольнике две стороны равны и параллельны, то этот четырехугольник будет параллелограммом.

Обоснование:

Рассмотрим квадрилатер ABCD. Пусть они смотрят на AB и CD параллельно. И пусть AB = CD. Проведем в нем диагональ BD. Он разделит этот четырехугольник на два равных треугольника: ABD и CBD.

Эти треугольники равны друг другу с двух сторон и угол между ними (BD — общая сторона, AB = CD по условию, angle1 = угол 2. Как пересечь нижележащие углы с помощью фиксирующего BD параллельно прямых AB и CD .), А значит, angle3 = angle4.

И эти углы будут ближе к пересечению прямых BC и AD, закрепляющих BD. Отсюда следует, что BC и AD параллельны друг другу. У четырехугольника ABCD противоположные стороны параллельны, а это означает, что четырехугольник ABCD является параллелограммом.

2 Знак параллелограмма

Если в четырехугольнике противоположные стороны попарно, то этот четырехугольник будет параллелограммом.

Обоснование:

Рассмотрим квадрилатер ABCD.Проведем в нем диагональ BD. Он разделит этот четырехугольник на два равных треугольника: ABD и CBD.

Эти два треугольника будут равны друг другу по трем сторонам (BD — общая сторона, AB = CD и BC = AD при условии). Из этого можно сделать вывод, что угол1 = угол2. Отсюда следует, что AB параллельна CD. А так как AB = CD и AB параллельны CD, то первой особенностью параллелограмма, квадрилатром ABCD будет параллелограмм.

3 Знак параллелограмма

Если диагонально пересекаются в четырехугольнике, а точка пересечения делится пополам, то этот четырехугольник будет параллелограммом.

Рассмотрим квадрилатер ABCD. Проведем в нем две диагонали AC и BD, которые пересекутся в точке O и разделят эту точку пополам.

треугольников AOB и COD будут равны друг другу по первому признаку равенства треугольников. (AO = OC, BO = OD по условию, угол AOB = COD угол как вертикальные углы.) Следовательно, AB = CD и angle1 = Angle 2. Из равенства углов 1 и 2 имеем, что AB параллельна CD. Тогда у нас есть, что в четырехугольнике ABCD сторона AB равна CD и параллельна, а на первом элементе параллелограмма четырехугольник ABCD будет параллелограммом.

Муниципальное бюджетное общеобразовательное учреждение

Савинская средняя общеобразовательная школа

Исследования

Параллелограмм и его новые свойства

Выполнила: ученица 8Б класса

МБОУ Савинская Сош

Кузнецова Светлана, 14 лет

Руководитель: Учитель математики

Тульчевская Н.А.

стр. Савино

Ивановская область, Россия

2016.

I. Введение ________________________________________________________________________

II. Из истории параллелограмма ___________________________________ стр. 4

III Дополнительные свойства параллелограмма ______________________ стр. 4

IV. Подтверждение свойств _____________________________________ стр. 5

V. Решение задач с использованием дополнительных свойств __________ стр. 8

Vi.Применение свойств параллелограмма в жизни ___________________ стр. 11

VII. Заключение _____________________________________________ стр. 12

VIII. Литература _____________________________________________ стр. 13

    Введение

« Среди равных умов

для эквити других условий

выше того, кто знает геометрию»

(Blaze Pascal).

При изучении темы «Параллелограммы» на уроках геометрии мы рассматривали два свойства параллелограмма и три знака, но когда начали решать задачи, оказалось, что этого недостаточно.

У меня вопрос, а есть ли у параллелограмма еще и свойства, и как они помогут при решении задач.

И я решил изучить дополнительные свойства параллелограмма и показать, как их можно применить для решения задач.

Предмет исследования
:
параллелограмм

Объект исследования
: Свойства поллограммы
Цель работы:

    формулировка и доказательство дополнительных свойств параллелограмма, которые не изучаются в школе;

    использование этих свойств для решения проблем.

Задач:

    Найти дополнительную литературу по изучаемой теме;

    Изучите дополнительные свойства параллелограмма и докажите их;

    Покажите использование этих свойств для решения проблем;

    Рассмотрим использование свойств параллелограмма в жизни.
    Методы исследования:

    Работа с учебной и научно-популярной литературой, ресурсами сети Интернет;

    Изучение теоретического материала;

    Выбор круга задач, которые можно решить с помощью дополнительных свойств параллелограмма;

    Наблюдение, сравнение, анализ, аналогия.

Продолжительность исследования
: 3 месяца: январь-март 2016 г.

    1. Из истории параллелограмма

В учебнике геометрии мы читаем следующее определение параллелограмма: Поллограмм — такой четырехугольник, у которого противоположные стороны попарно параллельны

Слово «параллелограмм» переводится как «параллельные прямые» (от греческих слов Parallelos — параллель и грамм — линия), этот термин ввел Евклид.В своей книге «Начинающий» Евклид доказал следующие свойства параллелограмма: противоположные стороны и углы параллелограмма равны, а диагональ делит его пополам.
О точке пересечения параллелограмм Евклида не упоминается. Только к концу средневековья была разработана полная теория параллелограммов и только в XVII веке в учебниках появились теоремы о параллелограммах, которые доказываются с помощью теоремы Евклида о свойствах параллелограмма.

III

Дополнительные свойства параллелограмма

В учебнике по геометрии приведены всего 2 свойства параллелограмма:

    Противоположные углы и стороны равны

    Диагональ параллелограмма пересекается, а точка пересечения делится пополам

В различных источниках геометрии можно найти следующие дополнительные свойства:

    Количество смежных углов Поллограмм, равный 180 0

    Биссектриса угла параллелограмма отсекает от него равносторонний треугольник;

    Биссектрисы противоположных углов параллелограмма лежат на параллельных прямых;

    Биссектриса смежных углов Параллелограмм пересекаются под прямыми углами;

    Биссектриса всех углов параллелограмма с пересечением образует прямоугольник;

    Расстояния от противоположных углов параллелограмма до одной и той же диагонали равны.

    Если в параллелограмме соединить противоположные вершины с серединой противоположных сторон, то получится другой параллелограмм.

    Сумма квадратов диагоналей параллелограмма равна двойной сумме квадратов его смежных сторон.

    Если в параллелограмме двух противоположных углов провести высоты, то получится прямоугольник.

IV

Доказательство свойств параллелограмма

    Количество смежных углов Параллелограмм равен 180
    0

Dano :

ABCD — параллелограмм

Prove

A +.
Б =.

Доказательства:

AI
B-изогнутые односторонние углы с параллельным прямым солнцем AD и Sale AV, то есть
A +.
Б =.

2

Дано: ABCD
параллелограмм,

АК-Биссектрис
НО.

Доказательство AVK — равно

Доказательства:

1)
1 =
3 (перекрестно проходит под солнцем AD и Sale AK)

2)
2 =
3 т.К. Ак — бисектриса,

Так 1 =
2.

3) АВК — нерешенный т. 2. 2 угла треугольника равны

. Биссектрисный угловой параллелограмм образует равносторонний треугольник

3

Дано: AVD — параллелограмм,

Ak — Bissektris A,

CP — Bissektris C.

Prove AK ║ ср.

Обоснование:

1) 1 = 2 т.К. Ак-bissectrice

2) 4 = 5 т.к. ср — Bissektris

3) 3 = 1 (Малые углы

Sun ║ AD и AK-SECOND),

4 ) A = C (по свойству параллелограмма), значит 2 = 3 = 4 = 5.

4) из пунктов 3 и 4 следует, что 1 = 4, и этим углам соответствуют для прямого АК и КП и Южного Солнца,

Итак, АК ║ КП (на основе параллельности прямого)

.Биссектриса противоположных углов Параллелограмм лежит на параллельных прямых

    Биссектриса смежных углов Параллелограмм пересекается под прямым углом

Дано: AVD — параллелограмм,

Ak-bisectaris A,

DR Bissectrix D

Prove DR AK.

Свидетельство:

1) 1 = 2, потому что Ak — Bissektris

Пусть, 1 = 2 = x, тогда a = 2x,

2) 3 = 4, потому что DR — Bissectrix

Пусть, 3 = 4 = y, тогда d = 2y

3) a + d = 180 0, т.к. сумма смежных углов Параллелограмма равна 180

2) Рассмотрим A OD.

1 + 3 = 90 0, тогда

5. Биссектрика всех углов Параллелограмм с пересечением образуют прямоугольник

Дано: AVD — параллелограмм, ak-bisectaris A,

DR, биссектриса D,

Cm -Bistektris C,

BF -Bissectrice b.

Доказательство : КРНС -ПРЯУГОН

Доказательство:

Исходя из предыдущего свойства 8 = 7 = 6 = 5 = 90 0,

так что КРНС — яркий.

    Расстояния от противоположных углов параллелограмма до одной и той же диагонали равны.

Дано: ABCD-параллелограмм, as-диагональ.

VC Ac DP. AC

Prove BC = DR

Evidence: 1) DCR = KAB, как внутренние замыкания, лежащие в основе AB ║ CD и SECOND AC.

2) АКБ =. CDR (сбоку и два регулировочных угла AV = CD Cd p = AB K).

А в равных треугольниках соответствующие стороны равны, значит DR = BC.

    Если в параллелограмме соединить противоположные вершины с серединой противоположных сторон, то получится другой параллелограмм.

Дано: ABCD-параллелограмм.

Докаж ВКДР — параллелограмм.

Свидетельство:

1) BR = KD (ad = bc, указывает на и p

делит эти партии пополам)

2) BP ║ KD (лежит на AD BC)

Если в противоположные стороны четырехугольника равны и параллельны, значит, этот четырехугольник -паралограмма.

    Если в параллелограмме двух противоположных углов провести высоты, то получится прямоугольник.

    Сумма квадратов диагоналей параллелограмма равна двойной сумме квадратов его смежных сторон.

Дано: ABCD — параллелограмм. BD и AC — диагональ.

Prove AC 2
+ тд. 2
= 2 (AB 2
+ AD 2
)

Доказательства: 1) СПРОСИТЬ:


AC
² =


+

2) Б.
R D.
:
BD.
2
=
Б.
R 2
+ Р. Д.
2
(по теореме Пифагора)

3 )
AC
² +
BD.
² = SCQ + A.
К² +. Б.
R² + R. D.
²

4) СК = БП = N (высота )

5) AC. 2
+ Б. Д.
2
=
H.
2
+
А.
К 2
+
H.
2
+ Р. Д.
2

6)
Пусть будет
Д.
К =. А.
P = H. , затем C.
К Д.
:
H.
2
=
CD
2
— Н. 2

согласно теореме Пифагора )

7) как xt + в D.
² = S. D.
2
— x² + ак 1
² +
CD
2
-H. 2
+ Р. Д.
2
, г.

As² + B. Д.
² = 2s D.
2
-2x 2
+
А.
К 2
+ Р. Д.
2

8) А.
К = AD +.
ч. , г.

R D = ad
ч. , г.

As² + B. D.
² = 2. CD
2
-2x 2
+ ( AD
+ х) 2
+ ( н.э.
) 2
, г.

AC ² +

IN D² = 2.
ИЗ D²-2
ч. ² + AD.
2

+ 2AD
ч. +

ч. 2

+ нашей эры
2

-2ад.
ч. +

ч. 2

,

AC ² +

IN D² = 2cd.
2

+ 2AD
2

= 2 (CD
2

+ нашей эры
2

).

В.

. Решение задач с использованием этих свойств

    Точка пересечения биссектрисы двух углов параллелограмма, примыкающих к одной стороне, принадлежит противоположной стороне. Меньшая сторона параллелограмма равна 5.
    . Найдите это с самой стороны.

Дано: ABCD — параллелограмм,

AK — Bissektris
BUT,

D K — биссектриса
D, av = 5

Найти : Sun.

беспорядок

Решение

Поскольку AK — Bissektris
A, то AVC является равным.

Потому что DK — биссектриса
D, T. DCK — Wireless

DC = CK = 5

Тогда Sun = VK + SK = 5 + 5 = 10

Ответ: 10.

2. Найдите периметр параллелограмма, если биссектриса одного из его углов делит сторону параллелограмма на отрезки 7 см и 14 см.

1 корпус

Дано:
НО,

ВК = 14 см, kc = 7 см

Найти: Параллелограмм

3 u0003

u0036

Decision Sun

VK + kc = 14 + 7 = 21 (см)

Поскольку AK — Bissektris
A, то AVC является равным.

AV = VK = 14 см

Тогда p = 2 (14 + 21) = 70 (см)

происходящее

Дано: ABCD — параллелограмм,

DK — биссектриса
D,

ВК = 14 см, kc = 7 см

Найти : P параллелограмм

Решение

Солнце = ВК + kc = 14 + 7 = 21 (см)

Потому что ДК — биссектриса
D, T.DCK — Wireless

DC = CK = 7

Тогда p = 2 (21 + 7) = 56 (см)

Ответ: 70см или 56 см

3. Детекторами параллелограмма являются равны 10 см и 3 см. Биссектриса двух углов, примыкающих к большей стороне, делит противоположную сторону на три сегмента. Найдите эти сегменты.

1 случай: биссектрисы пересекаются вне параллелограмма

Дано: ABCD — параллелограмм, ак — биссектриса
А,

DK — биссектриса
D, av = 3 см, солнце = 10 см

Найти : VM, Mn, NC

Решение

Поскольку AM — Bissektris
A, тогда AVM является равным.

Т.к. DN — Bissektris
D, T. DCN — равно

DC = CN = 3

Тогда Mn = 10 — (Bm + NC) = 10 — (3 + 3) = 4 см.

2 случай: бисектрис пересекаются внутри параллелограмма

Поскольку An — Bissektris
A, то Avn является равновесием.

АВ = Б. Н.
= 3
D.

А раздвижная сетка — для перемещения на необходимое расстояние в дверном проеме

Механизм параллелограмм — Механизм четырехжильный, звенья которого представляют собой параллелограммы.Применяется для реализации поступательного движения с помощью навесных механизмов.

Параллелограмм с неподвижным звеном — Одно звено неподвижно, противоположное совершает колебательное движение, оставаясь параллельным неподвижным. Два соединенных между собой параллелограмма дают конечному звену две степени свободы, оставляя его параллельным неподвижному.

Примеры: автобусы, погрузчики, треноги, подвески, автомобильные подвески.

Параллелограмм с фиксированным шарниром — свойство параллелограмма используется для поддержания постоянного отношения расстояний между тремя точками.Пример: Исходный пантограф — устройство для масштабирования чертежей.

Ромб — Все звенья одинаковой длины приближаются (затягиваются) Пара противоположных шарниров приводит к раскрытию двух других петель. Все ссылки работают на сжатие.

Примеры — автомобильный ромбовидный домкрат, трамвайный пантограф.

Sciser или Х-образный механизм , также известный как ножницы Nuremberg — вариант рома — два звена, соединенные в середине петли. Достоинства механизма — компактность и простота, недостаток — наличие двух пар скольжения.Два (или более) таких механизма, соединенных последовательно, образуют середину ромба (ов). Используется в слушателях, детских игрушках.

VII

Заключение

Кто с детства занимается математикой,

развивает внимание, тренирует свой мозг,

свою волю, повышает настойчивость

и настойчивость в достижении цели

Маркус А.

    В процессе работы я доказал дополнительные свойства параллелограмма.

    Я убедился, что применяя эти свойства, вы сможете быстрее решать поставленные задачи.

    Я показал, как эти свойства применяются на примерах решения конкретных задач.

    Я узнал много нового о параллелограмме, чего нет в нашем учебнике по геометрии

    Я убедился, что знание геометрии очень важно в жизни на примерах применения свойств параллелограмма .

Цель моей исследовательской работы выполнена.

Насколько важны математические знания, факт, что награда учреждена тому, кто издаст книгу о человеке, который жил без помощи математики. Эту премию еще не получил ни один человек.

VIII.

Литература

    1. Погорелова А.А. Геометрия 7-9: Учебник для общего образования. Учреждения-М.: Просвещение, 2014

      Л.С. Танасян и др. Геометрия. Дополнительный. К учебнику 8 кл: ученица. Пособие для школьников и классов с изображением. Исследования. Математика. — М .: Вита Пресс, 2003

      Интернет-ресурсы

      Материалы википедии

Тематический урок

  • Диагональный параллелограмм объекта.

Задачи урока

  • Познакомьтесь с новыми определениями и вспомните некоторые из уже изученных.
  • Сформулируйте и докажите свойство диагоналей параллелограмма.
  • Научитесь применять свойства фигур при решении задач.
  • Развивающие — развивают внимание учащихся, усидчивость, усидчивость, логическое мышление, математическую речь.
  • Образовательные — через урок воспитывать внимательное отношение друг к другу, прививать умение слушать товарищей, взаимное исполнение, самостоятельность.

Задачи урока

  • Проверить умение учащихся решать задачи.

План урока

  1. Введение.
  2. Повторение ранее изученного материала.
  3. Поллограмма, ее свойства и признаки.
  4. Примеры задач.
  5. Независимая проверка.

Введение

«Крупное научное открытие дает решение большой проблемы, но и в решении любой задачи есть зерно открытия».

Свойство противоположных сторон параллелограмма

У параллелограмма противоположные стороны равны.

Доказательства.

Пусть ABCD будет этим параллелограммом. И пусть его диагонали пересекаются в точке О.
Так как Δ AOB = Δ COD по первому основанию равенства треугольников (∠ aob = ∠ COD, как и вертикаль, AO = OC, DO = OB, по свойству диагональ параллелограмма), то AB = CD. Аналогично из равенства треугольников Вос и ДОА следует, что ВС = DA. Теорема доказана.

Свойство параллелограмма противоположных углов

У параллелограмма противоположные углы равны.

Доказательства.

Пусть ABCD будет этим параллелограммом. И пусть его диагонали пересекаются в точке О.
Из доказанного в теореме о свойствах противоположных сторон параллелограмм Δ ABC = δ CDA в трех сторонах (AB = CD, BC = DA из доказанного, AC равен общий). Из равенства треугольников следует, что ∠ ABC = ∠ CDA.
Также доказано, что ∠ dab = ∠ BCD, что следует из ∠ ABD = ∠ CDB. Теорема доказана.

Диагональный параллелограмм недвижимости

Диагональ параллелограмма пересекается, а точка пересечения делится пополам.

Доказательства.

Пусть ABCD будет этим параллелограммом. Проводим диагональ переменного тока. Отметим на нем середину O. На продолжение отрезка DO отложите отрезок OB 1, равный DO.
Согласно предыдущей теореме AB 1 CD — параллелограмм. Следовательно, направьте AB 1 параллельно DC. Но через точку A можно провести только один прямой параллельный DC. Итак, прямая AB 1 совпадает с прямой AB.
Это также доказывает, что BC 1 совпадает с BC. Итак, точка C совпадает с 1.Параллелограмм ABCD совпадает с параллелограммом AB 1 CD. Следовательно, диагональ параллелограмма пересекается, а точка пересечения делится пополам. Теорема доказана.

В учебниках для обычных школ (например, в Погорелове) доказано так: диагонали параллелограммов разделены на 4 треугольника. Рассмотрим одну пару и выясним — они равны: основания у них противоположные стороны, примыкающие к ней соответствующие углы равны вертикали с параллельными прямыми.То есть отрезки диагоналей попарно равны. Все.

Это все?
Выше доказано, что точка пересечения делит диагональ пополам — если есть. Само существование приведенных выше рассуждений никоим образом не доказывает. То есть часть теоремы «диагональный параллелограмм пересекается» остается недоказанной.

Забавно, что доказать эту часть намного сложнее. Это, кстати, следует из более общего результата: любая выпуклая четырехконтактная диагональ будет пересчитана, никаких неприятных не будет.

О равенстве треугольников по бокам и двух поправках к нему (второй признак равенства треугольников) и др.

Теорема о равенстве двух треугольников на стороне и двух смежных углов Фалеса нашла важное практическое применение. В гавани Милеты построили дальномер, определяющий расстояние от корабля до моря. Он представлял собой три сбитые сваи A, B и C (AB = Sun) и место прямого SC, перпендикулярного.Когда корабль появляется на прямом SC, была точка D такая, что точки d ,. In и e оказались на одной прямой. Как видно из рисунка, расстояние от КД на земле — это желаемое расстояние до корабля.

Вопросы

  1. Диагональ квадрата точки пересечения делится пополам?
  2. Диагональный параллелограмм равен?
  3. Управляющие углы параллелограмма равны?
  4. Слово определение параллелограмма?
  5. Сколько знаков у параллелограмма?
  6. Может ли ромб быть параллелограммом?

Список использованных источников

  1. Кузнецов А.В., учитель математики (5-9 классы), Киев
  2. «ЕГЭ 2006. Математика. Учебные материалы для обучения студентов / Рособрнадзор, Исоп — М .: Интеллект-Центр, 2006»
  3. Мазур К.И. «Решение основных соревновательных задач по математике сборника под ред. М.И. Сканави»
  4. Атанасян Л.С., Бутузов В.Ф., Кадомцев С.Б., Позняк Е.Г., Юдина И.И. «Геометрия, 7 — 9: Учебник для общеобразовательных учреждений»

Над уроком работали

Кузнецов А.V.

Purknak S.A.

Евгений Петров

Задать вопрос о современном образовании, выразить идею или решить уреранную проблему можно. Образовательный форум Где на международном уровне собирается образовательный совет свежих мыслей и действий. Создание blog Вы не только повысите свой статус грамотного учителя, но и внесете весомый вклад в развитие школы будущего. Гильдия лидеров образования Открывает двери для высококлассных специалистов и приглашает к сотрудничеству в направлении создания лучших школ мира.

Предмет> Математика> Математика 8 класс

В параллелограмме со всех сторон. Параллелограмм и его свойства

Параллелограмм называется четырехугольником, у которого противоположные стороны параллельны, т.е. лежат на параллельной прямой

Свойства Pollogram:
Теорема 22.
Противоположные стороны параллельного грамма равны.
Доказательства. В параллелограмме АВД проведем диагональ АК. Треугольники ACD и DCA равны, так как имеют общую сторону переменного тока и две пары равных углов.На него отправлено: ∠ Sav = ∠ ACD, ∠ DC = ∠ DAC (как нижележащие углы при параллельных прямых AD и Sun). Итак, Av = CD и Sun = AD, как соответствующие стороны равных треугольников, гл.Т.Д. Из равенства этих треугольников следует также равенство соответствующих углов треугольников:
Теорема 23.
Противоположные углы параллелограмма равны: ∠ A = ∠ C и ∠ B = ∠ D.
Равенство первой пары происходит из равенства треугольников AVD и CBD, а второй — ABC и ACD.
Теорема 24.
Соседние углы параллелограмма, т.е. углы, прилегающие к одной стороне, составляют 180 градусов.
Это потому, что это внутренние односторонние углы.
Теорема 25.
Диагональ параллелограмма должна делиться друг на друга в точке их пересечения пополам.
Доказательства. Рассмотрим треугольники Bos и AOD. Согласно первому свойству AD = Sun ∠ OAD = ∠ ASS и ∠ ODA = ∠ ABS, поскольку лежит в основе под параллельными прямыми AD и Sun.Следовательно, треугольники ВС и АОП равны по бокам и прилегающим к ним углам. Итак, при = OD и AO = OS, как соответствующие стороны равных треугольников, гл.Т.Д.

Знаки параллелограмма
Теорема 26.
Если противоположные стороны четырехугольника попарно равны, то это параллелограмм.
Доказательства. Предположим, что в четырехугольнике AVD есть AD и Sun, AV и CD соответственно (Riga2). Проведем диагональ динамиков. Треугольники ABC и ACD равны трем сторонам.Тогда углы вас и DSA равны и, следовательно, автоматически параллельны CD. Параллельность сторон Солнца и AD следует из равенства углов CAD и DR.
Теорема 27.
Если противоположные углы четырехугольника попарно равны, то это параллелограмм.
Пусть ∠ A = ∠ C и ∠ B = ∠ D. Поскольку ∠ A + ∠ B + ∠ C + ∠ D = 360 O, то ∠ A + ∠ B = 180 O и сторона AD и солнце параллельны ( на основе параллелизма прямой). Мы также доказываем параллелизм сторон AV и CD и заключаем, что AVD является параллелограммом по определению.
Теорема 28.
Если прилегающие углы четырехугольника, т.е. углы, прилегающие к одной стороне, составляют 180 градусов, это параллелограмм.
Если внутренние односторонние углы в сумме равны 180 градусам, то прямолинейно быстро. Итак, AV paralo CD и sun paral ad. Четырехугольник по определению оказывается параллелограммом.
Теорема 29.
Если диагональ четырехугольника в точке пересечения пополам делится пополам, то четырехугольник является параллелограммом.
Доказательства. Если AO = OS, IC = OD, то треугольники AOD и VOS равны, так как имеют равные углы (по вертикали) вверху o, заключенного между парами равных сторон. Из равенства треугольников заключаем, что AD и Sun равны. Также равны стороны AB и CD, и четырехугольник оказывается параллелограммом на основании 1.
Теорема 30.
Если у четырехугольника есть пара равных параллельных сторон, то это параллелограмм.
Предположим, что у квадрилатера AVD стороны AB и CD параллельны и равны.Проводим диагонали АК и КД. Исходя из параллельности этих прямых, равенство должно быть равным нижележащим углам AVO = CDO и VAO = OSD. Треугольники AVO и CDO равны по бокам и примыкают к его углам. Следовательно, АО = ОС, I = OD, т.е. диагональ точки пересечения делится пополам и четырехугольник получается параллелограммом на основании 4.

В геометрии бывают частные случаи параллелограмм.

Тематический урок

  • Диагональный параллелограмм объекта.

Задачи урока

  • Познакомьтесь с новыми определениями и вспомните некоторые из уже изученных.
  • Сформулируйте и докажите свойство диагоналей параллелограмма.
  • Научитесь применять свойства фигур при решении задач.
  • Развивающие — развивают внимание учащихся, усидчивость, усидчивость, логическое мышление, математическую речь.
  • Образовательные — через урок воспитывать внимательное отношение друг к другу, прививать умение слушать товарищей, взаимное исполнение, самостоятельность.

Задачи урока

  • Проверить умение учащихся решать задачи.

План урока

  1. Введение.
  2. Повторение ранее изученного материала.
  3. Поллограмма, ее свойства и признаки.
  4. Примеры задач.
  5. Независимая проверка.

Введение

«Крупное научное открытие дает решение большой проблемы, но и в решении любой задачи есть зерно открытия.»

Свойство противоположных сторон параллелограмма

У параллелограмма противоположные стороны равны.

Доказательства.

Пусть ABCD будет этим параллелограммом. И пусть его диагонали пересекаются в точке О.
Так как Δ AOB = Δ COD по первому основанию равенства треугольников (∠ aob = ∠ COD, как и вертикаль, AO = OC, DO = OB, по свойству диагональ параллелограмма), то AB = CD.Аналогично из равенства треугольников Вос и ДОА следует, что ВС = DA. Теорема доказана.

Свойство параллелограмма противоположных углов

У параллелограмма противоположные углы равны.

Доказательства.

Пусть ABCD будет этим параллелограммом. И пусть его диагонали пересекаются в точке О.
Из доказанного в теореме о свойствах противоположных сторон параллелограмм Δ ABC = δ CDA в трех сторонах (AB = CD, BC = DA из доказанного, AC равен общий).Из равенства треугольников следует, что ∠ ABC = ∠ CDA.
Также доказано, что ∠ dab = ∠ BCD, что следует из ∠ ABD = ∠ CDB. Теорема доказана.

Диагональный параллелограмм недвижимости

Диагональ параллелограмма пересекается, а точка пересечения делится пополам.

Доказательства.

Пусть ABCD будет этим параллелограммом. Проводим диагональ переменного тока. Отметим на нем середину O. На продолжение отрезка DO отложите отрезок OB 1, равный DO.
Согласно предыдущей теореме AB 1 CD — параллелограмм. Следовательно, направьте AB 1 параллельно DC. Но через точку A можно провести только один прямой параллельный DC. Итак, прямая AB 1 совпадает с прямой AB.
Это также доказывает, что BC 1 совпадает с BC. Итак, точка C совпадает с 1. Параллелограмм ABCD совпадает с параллелограммом AB 1 CD. Следовательно, диагональ параллелограмма пересекается, а точка пересечения делится пополам. Теорема доказана.

В учебниках для обычных школ (например, в Погорелове) доказано так: диагонали параллелограммов разделены на 4 треугольника.Рассмотрим одну пару и выясним — они равны: основания у них противоположные стороны, примыкающие к ней соответствующие углы равны вертикали с параллельными прямыми. То есть отрезки диагоналей попарно равны. Все.

Это все?
Выше доказано, что точка пересечения делит диагональ пополам — если есть. Само существование приведенных выше рассуждений никоим образом не доказывает. То есть часть теоремы «диагональный параллелограмм пересекается» остается недоказанной.

Забавно, что доказать эту часть намного сложнее. Это, кстати, следует из более общего результата: любая выпуклая четырехконтактная диагональ будет пересчитана, никаких неприятных не будет.

О равенстве треугольников по бокам и двух поправках к нему (второй признак равенства треугольников) и др.

Теорема о равенстве двух треугольников на стороне и двух смежных углов Фалеса нашла важное практическое применение.В гавани Милеты построили дальномер, определяющий расстояние от корабля до моря. Он представлял собой три сбитые сваи A, B и C (AB = Sun) и место прямого SC, перпендикулярного. Когда корабль появляется на прямом SC, была точка D такая, что точки d ,. In и e оказались на одной прямой. Как видно из рисунка, расстояние от КД на земле — это желаемое расстояние до корабля.

Вопросы

  1. Диагональ квадрата точки пересечения делится пополам?
  2. Диагональный параллелограмм равен?
  3. Управляющие углы параллелограмма равны?
  4. Слово определение параллелограмма?
  5. Сколько знаков у параллелограмма?
  6. Может ли ромб быть параллелограммом?

Список использованных источников

  1. Кузнецов А.В., учитель математики (5-9 классы), Киев
  2. «ЕГЭ 2006. Математика. Учебные материалы для обучения студентов / Рособрнадзор, Исоп — М .: Интеллект-Центр, 2006»
  3. Мазур К.И. «Решение основных соревновательных задач по математике сборника под ред. М.И. Сканави»
  4. Атанасян Л.С., Бутузов В.Ф., Кадомцев С.Б., Позняк Е.Г., Юдина И.И. «Геометрия, 7 — 9: Учебник для общеобразовательных учреждений»

Над уроком работали

Кузнецов А.V.

Purknak S.A.

Евгений Петров

Задать вопрос о современном образовании, выразить идею или решить уреранную проблему можно. Образовательный форум Где на международном уровне собирается образовательный совет свежих мыслей и действий. Создание blog Вы не только повысите свой статус грамотного учителя, но и внесете весомый вклад в развитие школы будущего. Гильдия лидеров образования Открывает двери для высококлассных специалистов и приглашает к сотрудничеству в направлении создания лучших школ мира.

Предмет> Математика> Математика 8 класс

Доказательства

В первую очередь проведем диагональ АС. Получаются два треугольника: ABC и ADC.

Так как ABCD — параллелограмм, то верно следующее:

AD || ВС \ Стрелка вправо \ Угол 1 = \ Угол 2 Как лежащий крест.

AB || CD \ RIGHTARROW \ Angle3 = \ Angle 4 Как лежащий крест.

Следовательно, \ треугольник ABC = \ треугольник ADC (по второму основанию: и AC вообще).

А, значит, \ треугольник ABC = \ треугольник ADC, затем AB = CD и AD = BC.

Проверено!

2. Противоположные углы идентичны.

Свидетельство

Согласно свидетельству свойства 1. Мы знаем, что \ Угол 1 = \ Угол 2, \ Угол 3 = \ Угол 4. Таким образом, сумма противоположных углов равна равным: \ Угол 1 + \ Угол 3 = \ Угол 2 + \ Угол 4. Учитывая, что \ треугольник ABC = \ ТРЕУГОЛЬНИК получается АЦП \ угол а = \ угол с, \ \ угол b = \ угол d.

Проверено!

3. Диагональ отделяется половиной точки пересечения.

Доказательства

Проведем еще диагональ.

По свойство 1. Мы знаем, что противоположные стороны идентичны: AB = CD. Еще раз отметим поперечины, лежащие под равными углами.

Таким образом, видно, что \ треугольник aob = \ треугольник ячейка согласно второму признаку равенства треугольников (два угла и сторона между ними).То есть BO = OD (напротив углов \ angle 2 и \ angle 1) и AO = OC (напротив углов \ Angle 3 и \ Angle 4 соответственно).

Проверено!

Знаки параллелограмма

Если в вашей задаче присутствует только одна функция, фигура представляет собой параллелограмм и могут использоваться все свойства этой фигуры.

Для лучшего запоминания отметим, что знак параллелограмма ответит на следующий вопрос — «как узнать?» .То есть как узнать, что указанная фигура — параллелограмм.

1. Параллелограмм — это такой четырехугольник, в котором две стороны равны и параллельны.

AB = CD; AB || CD \\ RIGHTARROW ABCD — параллелограмм.

Доказательства

Рассмотрим больше. Почему реклама || ДО Н.Э?

\ треугольник ABC = \ треугольник АЦП ПО свойство 1. : AB = CD, AC — сумма и \ угол 1 = \ угол 2 как сшивки под параллелью AB и CD и ВТОРОЙ AC.

Но если \ треугольник ABC = \ треугольник ADC, то \ угол 3 = \ угол 4 (лежат напротив AB и CD соответственно). И поэтому ad || Bc (\\ Угол 3 и \\ Угол 4 — Лежащая свобода также равна).

Первый знак верный.

2. Параллелограмм — это такой четырехугольник, который равен противоположным направлениям.

AB = CD, ad = BC \ Rightarrow ABCD — параллелограмм.

Доказательства

Учтите эту особенность.Еще раз диагональ переменного тока.

По свойство 1. \ треугольник ABC = \ треугольник ACD.

Отсюда следует, что: \\ ANGLE 1 = \\ ANGLE 2 \\ RIGHTARROW AD || ДО Н.Э. и \\ УГОЛ 3 = \\ УГОЛ 4 \\ СТРЕЛКА ВПРАВО AB || CD, то есть ABCD — параллелограмм.

Второй знак верен.

3. Параллелограмм — это такой четырехугольник, в котором противоположные углы равны.

\ угол a = \ угол c, \ ANGLE B = \ ANGLE D \ RIGHTARROW ABCD — параллелограмм.(\\ CIRC) говорит, что AD || ДО Н.Э.

При этом \\ Альфа и \\ Бета — внутренние односторонние под Продажу AD. А это значит AB || CD.

Третий знак верен.

4. Параллелограмм — это такой четырехугольник, который по диагонали разделен точкой пересечения пополам.

АО = ОС; Бо = OD \ правый параллелограмм.

Доказательства

БО = OD; AO = OC, \ Angle 1 = \ Angle 2 как вертикаль \ Rightarrow \ треугольник aob = \ треугольник cod, \ Rightarrow \ Angle 3 = \ Angle 4, и \ Rightarrow AB || CD.

Аналогично БО = OD; AO = OC, \ ANGLE 5 = \ ANGLE 6 \ RIGHTARROW \ TRIANGLE AOD = \ TRIANGLE BOC \ RIGHTARROW \ ANGLE 7 = \ ANGLE 8, и \ rightarrow ad || ДО Н.Э.

Четвертый знак — верный.

1. Определение параллелограмма.

Если пара параллельных прямых пересекает другую пару параллельных прямых, то мы получаем четырехугольник, у которого противоположные стороны попарно параллельны.

В квадрилях ABDS и EFNM (рис.224) cd || AC и AB || CD;

EF || Mn и Em || Fn.

Квадрикон, противоположные стороны которого параллельны, называется параллелограммом.

2. Свойства параллелограмма.

Теорема. Диагональ параллелограмма делит его на два равных треугольника.

Предположим, есть параллелограммы ABDC (рис. 225), в которых AB || CD и AS || CD.

Требуется доказать, что диагональ делит ее на два равных треугольника.

Рисуем в параллелограмме ABDS Диагональ SV. Докажем, что \ (\ Delta \) Cab = \ (\ Delta \) SDV.

Сторона у этих треугольников общая; ∠abc = ∠BCD, как внутреннее прохождение нижележащих углов с параллельными AB и CD и Singing SV; ∠Acb = ∠SVD, также как внутренний проход нижележащих углов с параллельными переменным и cd и вековым CB.

Отсюда \ (\ Delta \) Cab = \ (\ Delta \) SDV.

Таким же образом можно доказать, что диагональ AD разделит параллелограммы на два равных треугольника ACD и ABD.

Следствие:

1
. Противоположные углы параллелограмма равны друг другу.

∠A = ∠D, следует из равенства треугольников CAB и SDV.

Аналогично и ∠С = ∠.

2. Противоположные стороны параллелограмма равны друг другу.

AB = CD и AC = CD, так как это стороны равных треугольников и лежат против равных углов.

Теорема 2. Диагональ параллелограмма в точке их пересечения делится пополам.

Пусть BC и AD — диагональ параллелограмма AVDS (рис. 226). Докажем, что АО = OD и CO = OB.

Для этого сравним пару противоположно расположенных треугольников, например \ (\ deelta \\) aob и \\ (\\ delta \) CD.

В этих треугольниках AB = Cd, как противоположные стороны параллелограмма;

∠1 = ∠2, так как углы внутреннего будут лежать под параллельными AB и CD и Single AD;

∠3 = ∠4 по той же причине, что и AB || CD и SV — их секанс.

Отсюда следует, что \ (\ Delta \) AOB = \ (\ Delta \) CD. А в равных треугольниках против равных углов есть равные стороны. Следовательно, АО = OD и CO = OB.

Теорема 3. Сумма углов, примыкающих к одной стороне параллелограмма, равна .
180 ° .

В параллелограмме ABCD проведем диагональ переменного тока и получим два треугольника ABC и ADC.

Треугольники равны, так как ∠1 = ∠4, ∠2 = ∠3 (нижележащие углы с параллельными прямыми), а сторона АС общая.
Из равенства \ (\ delta \) ABC = \ (\ Delta \) ADC следует, что AB = CD, BC = AD, ∠B = ∠D.

Сумма углов, прилегающих к одной стороне, например, углы A и D равны 180 ° как односторонние с параллельными прямыми линиями.

Муниципальное бюджетное образовательное учреждение

Савинская общеобразовательная школа

Научно-исследовательская

Параллелограмм и его новые свойства

Выполнила: ученица 8Б класса

МБОУ Савинская Сош

Кузнецова Светлана, 14 лет

Руководитель: Учитель математики

Тульчевская Н.А.

стр. Савино

Ивановская область, Россия

2016.

I. Введение ________________________________________________________________________

II. Из истории параллелограмма ___________________________________ стр. 4

III Дополнительные свойства параллелограмма ______________________ стр. 4

IV. Подтверждение свойств _____________________________________ стр. 5

V. Решение задач с использованием дополнительных свойств __________ стр. 8

Vi.Применение свойств параллелограмма в жизни ___________________ стр. 11

VII. Заключение _____________________________________________ стр. 12

VIII. Литература _____________________________________________ стр. 13

    Введение

« Среди равных умов

для эквити других условий

выше того, кто знает геометрию»

(Blaze Pascal).

При изучении темы «Параллелограммы» на уроках геометрии мы рассматривали два свойства параллелограмма и три знака, но когда начали решать задачи, оказалось, что этого недостаточно.

У меня вопрос, а есть ли у параллелограмма еще и свойства, и как они помогут при решении задач.

И я решил изучить дополнительные свойства параллелограмма и показать, как их можно применить для решения задач.

Предмет исследования
:
параллелограмм

Объект исследования
: Свойства поллограммы
Цель работы:

    формулировка и доказательство дополнительных свойств параллелограмма, которые не изучаются в школе;

    использование этих свойств для решения проблем.

Задач:

    Найти дополнительную литературу по изучаемой теме;

    Изучите дополнительные свойства параллелограмма и докажите их;

    Покажите использование этих свойств для решения проблем;

    Рассмотрим использование свойств параллелограмма в жизни.
    Методы исследования:

    Работа с учебной и научно-популярной литературой, ресурсами сети Интернет;

    Изучение теоретического материала;

    Выбор круга задач, которые можно решить с помощью дополнительных свойств параллелограмма;

    Наблюдение, сравнение, анализ, аналогия.

Продолжительность исследования
: 3 месяца: январь-март 2016 г.

    1. Из истории параллелограмма

В учебнике геометрии мы читаем следующее определение параллелограмма: Поллограмм — такой четырехугольник, у которого противоположные стороны попарно параллельны

Слово «параллелограмм» переводится как «параллельные прямые» (от греческих слов Parallelos — параллель и грамм — линия), этот термин ввел Евклид.В своей книге «Начинающий» Евклид доказал следующие свойства параллелограмма: противоположные стороны и углы параллелограмма равны, а диагональ делит его пополам.
О точке пересечения параллелограмм Евклида не упоминается. Только к концу средневековья была разработана полная теория параллелограммов и только в XVII веке в учебниках появились теоремы о параллелограммах, которые доказываются с помощью теоремы Евклида о свойствах параллелограмма.

III

Дополнительные свойства параллелограмма

В учебнике по геометрии приведены всего 2 свойства параллелограмма:

    Противоположные углы и стороны равны

    Диагональ параллелограмма пересекается, а точка пересечения делится пополам

В различных источниках геометрии можно найти следующие дополнительные свойства:

    Количество смежных углов Поллограмм, равный 180 0

    Биссектриса угла параллелограмма отсекает от него равносторонний треугольник;

    Биссектрисы противоположных углов параллелограмма лежат на параллельных прямых;

    Биссектриса смежных углов Параллелограмм пересекаются под прямыми углами;

    Биссектриса всех углов параллелограмма с пересечением образует прямоугольник;

    Расстояния от противоположных углов параллелограмма до одной и той же диагонали равны.

    Если в параллелограмме соединить противоположные вершины с серединой противоположных сторон, то получится другой параллелограмм.

    Сумма квадратов диагоналей параллелограмма равна двойной сумме квадратов его смежных сторон.

    Если в параллелограмме двух противоположных углов провести высоты, то получится прямоугольник.

IV

Доказательство свойств параллелограмма

    Количество смежных углов Параллелограмм равен 180
    0

Dano :

ABCD — параллелограмм

Prove

A +.
Б =.

Доказательства:

AI
B-изогнутые односторонние углы с параллельным прямым солнцем AD и Sale AV, то есть
A +.
Б =.

2

Дано: ABCD
параллелограмм,

АК-Биссектрис
НО.

Доказательство AVK — равно

Доказательства:

1)
1 =
3 (перекрестно проходит под солнцем AD и Sale AK)

2)
2 =
3 т.К. Ак — бисектриса,

Так 1 =
2.

3) АВК — нерешенный т. 2. 2 угла треугольника равны

. Биссектрисный угловой параллелограмм образует равносторонний треугольник

3

Дано: AVD — параллелограмм,

Ak — Bissektris A,

CP — Bissektris C.

Prove AK ║ ср.

Обоснование:

1) 1 = 2 т.К. Ак-bissectrice

2) 4 = 5 т.к. ср — Bissektris

3) 3 = 1 (Малые углы

Sun ║ AD и AK-SECOND),

4 ) A = C (по свойству параллелограмма), значит 2 = 3 = 4 = 5.

4) из пунктов 3 и 4 следует, что 1 = 4, и этим углам соответствуют для прямого АК и КП и Южного Солнца,

Итак, АК ║ КП (на основе параллельности прямого)

.Биссектриса противоположных углов Параллелограмм лежит на параллельных прямых

    Биссектриса смежных углов Параллелограмм пересекается под прямым углом

Дано: AVD — параллелограмм,

Ak-bisectaris A,

DR Bissectrix D

Prove DR AK.

Свидетельство:

1) 1 = 2, потому что Ak — Bissektris

Пусть, 1 = 2 = x, тогда a = 2x,

2) 3 = 4, потому что DR — Bissectrix

Пусть, 3 = 4 = y, тогда d = 2y

3) a + d = 180 0, т.к. сумма смежных углов Параллелограмма равна 180

2) Рассмотрим A OD.

1 + 3 = 90 0, тогда

5. Биссектрика всех углов Параллелограмм с пересечением образуют прямоугольник

Дано: AVD — параллелограмм, ak-bisectaris A,

DR, биссектриса D,

Cm -Bistektris C,

BF -Bissectrice b.

Доказательство : КРНС -ПРЯУГОН

Доказательство:

Исходя из предыдущего свойства 8 = 7 = 6 = 5 = 90 0,

так что КРНС — яркий.

    Расстояния от противоположных углов параллелограмма до одной и той же диагонали равны.

Дано: ABCD-параллелограмм, as-диагональ.

VC Ac DP. AC

Prove BC = DR

Evidence: 1) DCR = KAB, как внутренние замыкания, лежащие в основе AB ║ CD и SECOND AC.

2) АКБ =. CDR (сбоку и два регулировочных угла AV = CD Cd p = AB K).

А в равных треугольниках соответствующие стороны равны, значит DR = BC.

    Если в параллелограмме соединить противоположные вершины с серединой противоположных сторон, то получится другой параллелограмм.

Дано: ABCD-параллелограмм.

Докаж ВКДР — параллелограмм.

Свидетельство:

1) BR = KD (ad = bc, указывает на и p

делит эти партии пополам)

2) BP ║ KD (лежит на AD BC)

Если в противоположные стороны четырехугольника равны и параллельны, значит, этот четырехугольник -паралограмма.

    Если в параллелограмме двух противоположных углов провести высоты, то получится прямоугольник.

    Сумма квадратов диагоналей параллелограмма равна двойной сумме квадратов его смежных сторон.

Дано: ABCD — параллелограмм. BD и AC — диагональ.

Prove AC 2
+ тд. 2
= 2 (AB 2
+ AD 2
)

Доказательства: 1) СПРОСИТЬ:


AC
² =


+

2) Б.
R D.
:
BD.
2
=
Б.
R 2
+ Р. Д.
2
(по теореме Пифагора)

3 )
AC
² +
BD.
² = SCQ + A.
К² +. Б.
R² + R. D.
²

4) СК = БП = Н (высота )

5) AC. 2
+ Б. Д.
2
=
H.
2
+
А.
К 2
+
H.
2
+ Р. Д.
2

6)
Пусть будет
Д.
К =. А.
P = H. , затем C.
К Д.
:
H.
2
=
CD
2
— Н. 2

согласно теореме Пифагора )

7) как xt + в D.
² = S. D.
2
— x² + ак 1
² +
CD
2
-H. 2
+ Р. Д.
2
, г.

As² + B. Д.
² = 2s D.
2
-2x 2
+
А.
К 2
+ Р. Д.
2

8) А.
К = AD +.
ч. , г.

R D = ad
ч. , г.

As² + B. D.
² = 2. CD
2
-2x 2
+ ( н.э.
+ х) 2
+ ( н.э.
) 2
, г.

AC ² +

IN D² = 2.
ИЗ D²-2
ч. ² + AD.
2

+ 2AD
ч. +

ч. 2

+ нашей эры
2

-2ад.
ч. +

ч. 2

,

AC ² +

IN D² = 2cd.
2

+ 2AD
2

= 2 (CD
2

+ нашей эры
2

).

В.

. Решение задач с использованием этих свойств

    Точка пересечения биссектрисы двух углов параллелограмма, примыкающих к одной стороне, принадлежит противоположной стороне. Меньшая сторона параллелограмма равна 5.
    . Найдите это с самой стороны.

Дано: ABCD — параллелограмм,

AK — Bissektris
BUT,

D K — биссектриса
D, av = 5

Найти : Sun.

беспорядок

Решение

Поскольку AK — Bissektris
A, то AVC является равным.

Потому что DK — биссектриса
D, T. DCK — Wireless

DC = CK = 5

Тогда Sun = VK + SK = 5 + 5 = 10

Ответ: 10.

2. Найдите периметр параллелограмма, если биссектриса одного из его углов делит сторону параллелограмма на отрезки 7 см и 14 см.

1 корпус

Дано:
НО,

ВК = 14 см, kc = 7 см

Найти: Параллелограмм

3 u0003

u0036

Decision Sun

VK + kc = 14 + 7 = 21 (см)

Поскольку AK — Bissektris
A, то AVC является равным.

AV = VK = 14 см

Тогда p = 2 (14 + 21) = 70 (см)

происходящее

Дано: ABCD — параллелограмм,

DK — биссектриса
D,

ВК = 14 см, kc = 7 см

Найти : P параллелограмм

Решение

Солнце = ВК + kc = 14 + 7 = 21 (см)

Потому что ДК — биссектриса
D, T.DCK — Wireless

DC = CK = 7

Тогда p = 2 (21 + 7) = 56 (см)

Ответ: 70см или 56 см

3. Детекторами параллелограмма являются равны 10 см и 3 см. Биссектриса двух углов, примыкающих к большей стороне, делит противоположную сторону на три сегмента. Найдите эти сегменты.

1 случай: биссектрисы пересекаются вне параллелограмма

Дано: ABCD — параллелограмм, ак — биссектриса
А,

DK — биссектриса
D, av = 3 см, солнце = 10 см

Найти : VM, Mn, NC

Решение

Поскольку AM — Bissektris
A, тогда AVM является равным.

Т.к. DN — Bissektris
D, T. DCN — равно

DC = CN = 3

Тогда Mn = 10 — (Bm + NC) = 10 — (3 + 3) = 4 см.

2 случай: бисектрис пересекаются внутри параллелограмма

Поскольку An — Bissektris
A, то Avn является равновесием.

АВ = Б. Н.
= 3
D.

А раздвижная сетка — для перемещения на необходимое расстояние в дверном проеме

Механизм параллелограмм — Механизм четырехжильный, звенья которого представляют собой параллелограммы.Применяется для реализации поступательного движения с помощью навесных механизмов.

Параллелограмм с неподвижным звеном — Одно звено неподвижно, противоположное совершает колебательное движение, оставаясь параллельным неподвижным. Два соединенных между собой параллелограмма дают конечному звену две степени свободы, оставляя его параллельным неподвижному.

Примеры: автобусы, погрузчики, треноги, подвески, автомобильные подвески.

Параллелограмм с фиксированным шарниром — свойство параллелограмма используется для поддержания постоянного отношения расстояний между тремя точками.Пример: Исходный пантограф — устройство для масштабирования чертежей.

Ромб — Все звенья одинаковой длины приближаются (затягиваются) Пара противоположных шарниров приводит к раскрытию двух других петель. Все ссылки работают на сжатие.

Примеры — автомобильный ромбовидный домкрат, трамвайный пантограф.

Ножницы или Х-образный механизм , также известный как Нюрнбергские ножницы — вариант рома — два звена, соединенные в середине петли. Достоинства механизма — компактность и простота, недостаток — наличие двух пар скольжения.Два (или более) таких механизма, соединенных последовательно, образуют середину ромба (ов). Используется в слушателях, детских игрушках.

VII

Заключение

Кто с детства занимается математикой,

развивает внимание, тренирует свой мозг,

свою волю, повышает настойчивость

и настойчивость в достижении цели

Маркус А.

    В процессе работы я доказал дополнительные свойства параллелограмма.

    Я убедился, что применяя эти свойства, вы сможете быстрее решать поставленные задачи.

    Я показал, как эти свойства применяются на примерах решения конкретных задач.

    Я узнал много нового о параллелограмме, чего нет в нашем учебнике по геометрии

    Я убедился, что знание геометрии очень важно в жизни на примерах применения свойств параллелограмма .

Цель моей исследовательской работы выполнена.

Насколько важны математические знания, факт, что награда учреждена тому, кто издаст книгу о человеке, который жил без помощи математики. Эту премию еще не получил ни один человек.

VIII.

Литература

    1. Погорелова А.А. Геометрия 7-9: Учебник для общего образования. Учреждения-М.: Просвещение, 2014

      Л.С. Танасян и др. Геометрия. Дополнительный. К учебнику 8 кл: ученица. Пособие для школьников и классов с изображением. Исследования. Математика. — М .: Вита Пресс, 2003

      Интернет-ресурсы

      Материалы википедии

Средняя линия треугольника. Теорема Фалеса. Полные уроки

О параллельном и секущем.

Вне русскоязычной литературы о теореме Фалеса иногда ссылаются на другую теорему о планиметре, а именно на утверждение, что вписанный угол, основанный на диаметре окружности, является прямым.Открытие этой теоремы действительно приписывается Фалесу, что является свидетельством лагеря.

Состав

Если один из двух непосредственно отложить последовательно несколько равные отрезки и через их концы провести параллельную прямую, пересекающую вторую прямую, то они отрежут ко вторым прямым равным отрезкам.

Более общая формулировка, также называемая теоремой о пропорциональных сегментах

Параллельная прямая линия, отрезанная на последовательных пропорциональных сегментах:

A 1 A 2 B 1 B 2 = A 2 A 3 B 2 B 3 = A 1 A 3 B 1 B 3.(\ DisplayStyle (\\ FRAC (A_ (1) A_ (2)) (b_ (1) b_ (2))) = (\ FRAC (A_ (2) A_ (3)) (B_ (2) B_ (3))) = (\ FRAC (A_ (1) A_ (3)) (b_ (1) b_ (3))).)

Замечания

  • Теорема не имеет ограничений на взаимное расположение секущей (верно как для пересекающихся прямых, так и для параллельных). Также не имеет значения, где находятся отрезки на секущей.
  • Теорема Фалеса является частным случаем теоремы о пропорциональных сегментах, поскольку равные сегменты могут считаться пропорциональными сегментами с коэффициентом пропорциональности 1.

Доказательство в случае последовательного

Рассмотрим вариант с несвязанными парами отрезков: пусть угол пересекает прямую A A 1 | | B B 1 | | C C 1 | | D d 1 (\ displaystyle aa_ (1) || bb_ (1) || cc_ (1) || dd_ (1)), где A B = C D (\ DisplayStyle AB = CD).

Доказательство в случае параллельной прямой

Проведем прямую до н.э. . Уголки ABC, и BCD. равны внутренним шкафам, лежащим в основе параллельных прямых AB, и CD, и Sale BC., и уголки ACB. и CBD. равно внутренним шкафам, лежащим в основе параллельными прямыми AC и BD. и продажа до н.э. . Затем по второму знаку равенства треугольников треугольники ABC, и DCB. равных. Отсюда следует, что AC = BD. и AB = CD . ■

Варианты и обобщения

Обратная теорема

Если есть равные отрезки в теореме Фалеса (часто в школьной литературе используется эта формулировка) обратная теорема Также оказывается верной.Для пересекающихся секверов он формулируется как:

В теореме обратного Фаблеса важно, чтобы равные сегменты начинались с вершины

Таким образом (см. Рис.) Из чего CB 1 Ca 1 = B 1 B 2 A 1 A 2 = … (\ displayStyle (\ FRAC (CB_ (1)) (Ca_ (1))) \ u003d (\ FRAC (B_ (1) B_ (2)) (A_ (1) A_ (2))) = \ ldots), следует, что A 1 B 1 | | A 2 B 2 | | … (\\ displaystyle a_ (1) b_ (1) || a_ (2) b_ (2) || \\ ldots).

Если последовательность параллельна, то необходимо потребовать равенства отрезков на обеих секантиях между собой, иначе это утверждение станет некорректным (контрпример — трапеция, пересекаемая линией, проходящей через середину оснований).

Эта теорема используется в навигации: столкновение судов, движущихся с постоянной скоростью, неизбежно, если сохраняется направление от одного судна к другому.

Лемма Соллертинский

Следующее утверждение, двойственное лемме Соллертина:

Пусть будет F (\ DisplayStyle F) — проективное соответствие между точками прямого L (\ DisplayStyle L) и прямого M (\ DisplayStyle M). Тогда набор прямых будет набором касательных к некоторому коническому сечению (возможно, вырожденному).

В случае теоремы фаз коника будет бесконечно удаленной точкой, соответствующей направлению параллельных прямых.

Это утверждение, в свою очередь, является крайним случаем следующего утверждения:

Пусть будет F (\ DisplayStyle F) — Проективное преобразование хвойных пород. Тогда конверт множества прямых X F (X) (\\ DisplayStyle XF (X)) Будет коника (возможно, вырожденная).

Гробница небольшая, но слава о ней безмерна.
Перед вами много разумных фантазий.

Надпись на могиле Фалеза Мирецкого

Представьте себе такую ​​картинку. 600 г. до н. Э. Египет. Перед вами огромная египетская пирамида. Чтобы удивить фараона и остаться в его фаворитах, вам нужно измерить высоту этой пирамиды. В вашем распоряжении … ничего. Можно впасть в отчаяние, а можно сделать так, как Фалез Милецкий : Используйте подобие треугольников.Да, оказывается, все достаточно просто. Фалез Милецкий дождался совпадения длины его тени и ее роста, а затем с помощью теоремы о подобии треугольников нашел длину тени пирамиды, которая, соответственно, была равна отбрасываемой тени пирамиды.

Кто этот Фалез Милецкий ? Человек, снискавший славу одного из «семи мудрецов» древности? Фалез Мирецкий — древнегреческий философ, отличившийся успехами в области астрономии, а также математики и физики.Годы его жизни были установлены лишь приблизительно: 625-645 GG BC

Среди свидетельств знания астрономии Фалеса можно привести следующий пример. 28 мая 585 г. до н.э. Милецкое предсказание солнечного затмения Помогло остановить последние 6 лет войны между Лидией и MiDius. Это явление настолько напугало Миддов, что они согласились на невыгодные условия заключения мира с лидийцами.

Достаточно широко известна легенда, характеризующая Фалеса как находчивого человека.Фалесу часто приходилось слышать нелестные отзывы о его бедности. Однажды он решил доказать факт, что оба философа при желании могут жить в достатке. Еще зимой Фалес для наблюдения за звездами определил, что летом будет хороший урожай Маслин. Потом нанял масломб в просо и на Хиосе. Обошлось ему довольно дешево, так как зимой спрос на них практически отсутствует. Когда оливки дали богатый урожай, Фалес стали раздавать свои маслодавилы. Собранные этим методом большие деньги были расценены как свидетельство того, что философы могут заработать своим умом, но их призвание выше таких земных проблем.Эту легенду, кстати, повторил сам Аристотель.

Что касается геометрии, то многие ее «открытия» были заимствованы у египтян. И все же такая передача знаний в Грецию считается одной из главных заслуг Фалеза Милецкого.

Достижения Фалеса считаются формулировкой и доказательством следующих теорем:

  • вертикальные углы равны;
  • равных треугольников признают те, у которых сторона и два смежных угла соответственно равны;
  • углы при основании равномерного треугольника равны;
  • Диаметр

  • делит круг пополам;
  • вписанный угол, основанный на диаметре, является прямым.

Именем Фалеса названа еще одна теорема, полезная при решении геометрических задач. Есть обобщенный и частный взгляд, обратная теорема, формулировки тоже могут немного отличаться в зависимости от источника, но смысл их у всех остается единым. Рассмотрим эту теорему.

Если параллельные прямые пересекают сторону угла и отсекают с одной стороны равные отрезки, то отрезают равные отрезки и с другой его стороны.

Предположим, точки A 1, a 2 и 3 — точки пересечения параллельных прямых с одной из сторон угла, а в 1, 2 и 3 точки пересечения параллельных прямых с другой стороны угла. угол.Необходимо доказать, что если 1 и 2 = 2 А 3, то в 1 при 2 = в 2 в 3.

После точки в 2 проведем прямую, параллельную прямой 1 A 2. Обозначим новую прямую с 1 C 2. Рассмотрим параллелограмм A 1 C 1 B 2 A 2 и A 2 B 2 C 2 A 3.

Свойства параллелограмма позволяют утверждать, что A1A2 = C 1 B 2 и A 2 A 3 = B 2 C 2. А поскольку по нашему условию a 1 A 2 = a 2 A 3, то C 1 B 2 = в 2 c 2.

И, наконец, рассмотрим треугольники Δ C 1 B 2 B 1 и δ C 2 B 2 B 3.

C 1 B 2 = B 2 C 2 (доказано выше).


Это означает, что Δ C 1 B 2 B 1 и Δ C 2 B 2 B 3 будут равны на втором основании равенства треугольников (на боковых и прилегающих углах).

Таким образом, теорема Фалеса доказана.

Использование этой теоремы значительно облегчит и ускорит решение геометрических задач. Успехов в развитии этой занимательной математической науки!

сайт, при полном или частичном копировании материала ссылка на первоисточник обязательна.

Тематический урок

Задачи урока

  • Познакомьтесь с новыми определениями и вспомните некоторые из уже изученных.
  • Сформулируйте и докажите свойства квадрата, докажите его свойства.
  • Научитесь применять свойства фигур при решении задач.
  • Развивающие — развивают внимание учащихся, усидчивость, усидчивость, логическое мышление, математическую речь.
  • Образовательные — через урок воспитывать внимательное отношение друг к другу, прививать умение слушать товарищей, взаимное исполнение, самостоятельность.

Задачи урока

  • Проверить умение учащихся решать задачи.

План урока

  1. Историческая справка.
  2. Фалес как математик и его работы.
  3. Полезно запомнить.

Историческая справка

  • Теорема Фалеса все еще используется в морской навигации, как правило, что столкновение судов, движущихся с постоянной скоростью, неизбежно, если сохраняется курс суда.

  • Вне русскоязычной литературы о теореме Фалеса иногда ссылаются на другую теорему о планиметре, а именно на утверждение, что вписанный угол, основанный на диаметре окружности, является прямым. Открытие этой теоремы действительно приписывается Фалесу, что является свидетельством лагеря.
  • «Основы геометрии», составленные в Египте.

Открытие и заслуга ее автора

А знаете ли вы, что Фалез Милецкий был одним из семи самых известных по тем временам мудреца Греции.Он основал ионическую школу. Идея, которую продвигал Фалес в этой школе, заключалась в единстве всех вещей. Мудрец считал, что есть единое начало, с которого все и происходит.

Огромная заслуга Фалеза Милецкого — создание научной геометрии. Этому великому учению удалось на основе измерения египетского искусства создать дедуктивную геометрию, в основе которой лежат общие причины.

Помимо огромных познаний в геометрии, Фалес также хорошо разбирался в астрономии. Эму впервые удалось предсказать полное солнечное затмение.Но произошло это не в современном мире, а в далеком 585 году, еще до нашей эры.

Фалес Миетецкий был человеком, который понял, что север можно точно определить по созвездию маленького медведя. Но это было не последнее его открытие, так как ему удалось определить продолжительность года, разбить его на триста шестьдесят пять дней, а также установить время равноденствия.

Фалес на самом деле был разносторонним и мудрым человеком. К тому же он прославился как прекрасным математиком, так и физиком, астрономом, еще он, как настоящий метеоролог, умел довольно точно предсказывать урожай оливок.

Но наиболее примечательно то, что Фалес никогда не ограничивался своими знаниями только в области науки и теории и всегда пытался закрепить доказательства своих теорий на практике. И самое интересное, что Великий Мудрец не был сосредоточен на какой-то области своих знаний, его интересы имели разные направления.

Имя Фалеса уже тогда стало именным для мудреца. Его значение и значение для Греции было так же велико, как имя Ломоносова для России.Конечно, его мудрость можно трактовать по-разному. Но можно сказать, что у него есть как изобретательность, так и практическая несовместимость, а также в какой-то мере продолжение.

Фалез Милецкий был прекрасным математиком, философом, астрономом, любил путешествовать, был купцом и предпринимателем, занимался торговлей, а также был хорошим инженером, дипломатом, призером и активно участвовал в политической жизни. .

Ему даже удалось определить высоту пирамиды с помощью посоха и тени.Так и было. В один солнечный день Фалес положил свой посох на границу, где кончалась тень от пирамиды. Далее он подождал, когда его душа выйдет из тени, он станет равным его росту, и измерил длину тени пирамиды. Вот так, казалось бы, просто Фалес определил высоту пирамиды и доказал, что длина одной тени связана с длиной другой тени, а высота пирамиды связана с высотой посоха. Что и поразило самого фараона Амасиса.

Благодаря Фалесу все известные на тот момент знания были переведены в область научных интересов. Он смог довести результаты до уровня, подходящего для научного потребления, выделив определенный комплекс понятий. А может быть, с помощью Фалеса началось последующее развитие античной философии.

Теорема Фалеса играет одну важную роль в математике. Она была известна не только в Древнем Египте и Вавилоне, но и в других странах и была почвой для развития математики.А в повседневной жизни, при строительстве зданий, сооружений, дорог и т. Д., Без теоремы Фалеса не обойтись.

Теорема Фалеса в культуре

Теорема Фалеса стала известной не только в математике, но и в культуре. Однажды аргентинская музыкальная группа Les Luthiers (Iz.) На суд зрителей представила песню, которая была посвящена знаменитой теореме. Участники Les Luthiers в своем видеоклипе специально на эту песню представили доказательства прямой теоремы для пропорциональных отрезков.

Вопросы

  1. Какие прямые называются параллельными?
  2. Где практически применима теорема Фалеса?
  3. Что говорит теорема Фалеса?

Список использованных источников

  1. Энциклопедия для детей. Т.11. Математика / Глава. Ксенова М.Д.-М .: Аванта +, 2001.
  2. «ЕГЭ 2006. Математика. Учебные материалы для обучения студентов / Рособрнадзор, Исоп — М.: Интеллект-Центр, 2006 »
  3. Атанасян Л.С., Бутузов В.Ф., Кадомцев С.Б., Позняк Е.Г., Юдина И.И.« Геометрия, 7 — 9: Учебник для общеобразовательных учреждений »

Предмет> Математика> Математика 8 класс

Теорема не имеет ограничений на взаимное расположение секущей (верно как для пересекающихся прямых, так и для параллельных). Также не имеет значения, где находятся отрезки на секущей.

Доказательство в случае прямого параллельного

Проведем прямой БК.Углы ABC и BCD равны, как внутренние подъемы, лежащие с параллельными прямыми AB и CD и Singing BC, а углы ACB и CBD равны как внутренние, так и более близкие под параллельными прямыми AC и BD и единицей BC. Тогда по второму признаку равенства треугольников aBC треугольники и DCB равны. Отсюда следует, что AC = BD и AB = CD. ■

Также существует теорема о пропорциональных отрезках :

Параллельная прямая линия, отрезанная на последовательных пропорциональных сегментах:

\ FRAC (A_1A_2) (B_1B_2) = \ FRAC (A_2A_3) (B_2B_3) = \ FRAC (A_1A_3) (B_1B_3).

Теорема Фалеса является частным случаем теоремы о пропорциональных сегментах, поскольку равные сегменты могут считаться пропорциональными сегментами с коэффициентом пропорциональности 1.


Обратная теорема

Если в теореме Фалеса равные отрезки начинаются от вершины (часто в школьной литературе используется такая формулировка), обратная теорема также будет верна. Для пересекающихся секверов он формулируется как:

Таким образом (см. Рис.) Из чего \\ FRAC (CB_1) (CA_1) = \\ FRAC (B_1B_2) (A_1A_2) = \ ldots = (\ rm IDEM) Отсюда следует, что прямой A_1b_1 || a_2b_2 || \\ ldots.

Если последовательность параллельна, то необходимо потребовать равенства отрезков на обеих секантиях между собой, иначе это утверждение станет некорректным (контрпример — трапеция, пересекаемая линией, проходящей через середину оснований).

Варианты и обобщения

Следующее утверждение, двойственное лемме Соллертина:

  • Теорема Фалеса все еще используется в морской навигации, как правило, что столкновение судов, движущихся с постоянной скоростью, неизбежно, если сохраняется курс суда.
  • За пределами русскоязычной литературы о теореме Фалеса иногда ссылаются на другую теорему о планиметре, а именно на утверждение, что вписанный угол, основанный на диаметре окружности, является прямым. Открытие этой теоремы действительно приписывается Фалесу, что является свидетельством лагеря.

Написать отзыв на статью «Теорема Фалеса»

Литература

  • Атанасян Л.С. и др. Геометрия 7-9. — ред. 3-й — м .: Просвещение, 1992.

Банкноты

см. Также

  • Теорема Фалеса об угле на основе диаметра окружности

Отрывок, характеризующий теорему Фалеса

— Ничего не думаю, просто не понимаю …
— Постой, Соня, ты все поймешь. Посмотрите, что он за человек. Вы не думаете плохо ни обо мне, ни о нем.
— Ни о ком не думаю. Но что мне делать?
Соня не отказалась от нежного тона, которым к ней обратилась Наташа.Мягким и жалким было выражение лица Наташи, бросалось в глаза лицо Сони.
«Наташа, — сказала она, — ты просила меня не разговаривать с тобой, я не говорила, теперь ты начал». Наташа, я ему не верю. К чему эта загадка?
— Опять, снова! — прервала Наташа.
— Наташа, я за тебя боюсь.
— Чего ты боишься?
«Боюсь, что ты погубишь себя», — решительно сказала Соня, сама испугавшись того, что сказала.
Лицо Наташи снова выражало гнев.
— А я буду стрелять, грузить, как можно скорее опустошу себя. Не ваше дело. Не ты, но мне будет плохо. Уходи, оставь меня. Ненавижу тебя.
— Наташа! — испугалась Соня.
— Ненавижу, ненавижу! И ты мой враг навсегда!
Наташа выбежала из комнаты.
Наташа больше не разговаривала с Соней и избегала этого. С таким же выражением возбужденного удивления и преступления она ходила по комнатам, принимая время для другого занятия и немедленно бросая их.
Как ни тяжело было Сони, но она, не поливая глаз, наблюдала за своей девушкой.
Накануне дня, в который должен был вернуться граф, Соня заметила, что Наташа все утро сидела у окна гостиной, как будто чего-то ждала и что она делала какой-то военный знак, кого Соня приняла за Анатолия.
Соня стала еще внимательнее приглядывать за своей девушкой и заметила, что Наташа все время обеда и вечера находилась в странном и неестественном состоянии (отвечала неплею на заданные ей вопросы, начинала и фраз не было, он все смеялся) .
После чая Соня увидела девочку-грабителя, которая ждала ее у дверей Наташи. Она скучала по ней, и, подслушав у двери, он узнал, что письмо было передано снова. И вдруг Сыну стало ясно, что у Наташи какой-то ужасный план на этот вечер. Соня ее сбила. Наташа ей не позволила.
«Она убежит с ним! Подумала Соня. Она на все способна. Ныне это было что-то особенно жалкое и решительное. Она плакала, прощаясь с дядей, — вспоминала Соня.Да, верно, она с ним бежит — но что мне делать? «Подумала Соня, вспоминая теперь те приметы, которые ясно доказывали, почему у Наташи было какое-то ужасное намерение. Что делать, писать Курагину, требуя от него объяснений? Но кто ему велит отвечать? Напишите пирру, как я просил князя Андрея в случае неудачи? … Но, может быть, на самом деле она уже отказала Блокам (вчера она отправила письмо Накл Марье). Дядя Нет! «Сказать Марии Дмитриевне, которая верила в Наташу, сома казалась ужасной.«Но так или иначе, — подумала Соня, стоя в темном коридоре: сейчас или никогда не пришло время доказать, что я помню блага своей семьи и люблю Николая. Нет, я не буду спать три ночи, и я буду не выходи из этого коридора, и я не выгоню его из этого, и я не дам свою форму обрушиться на их семью », — подумала она.

Анатолий недавно переехал в Долохов. План похищения Ростовой уже несколько дней был респектабельным и был подготовлен Долоховым, и в тот день, когда Соня, подслушав Наташу в дверях, решила его защитить, этот план должен был быть осуществлен.Наташа в десять часов вечера обещала зайти к Курагину на заднее крыльцо. Курагин должен был высадить ее в вареной тройке и отнести за 60 верст от Москвы до села Каменка, где была приготовлена ​​прессованная попа, вынужденная выдать их замуж. В Каменке и был готов стоять, который должен был вывезти их по Варшавской дороге и там они должны были скакать за границу на почте.
У Анатолия был паспорт, и запалае, и десять тысяч денег, взятых у сестры, и десять тысяч, занятых у Долохова.
Два свидетеля — решка, бывший союз, по которому играли Дулохова и Макарина, отставного гусара, добродушного и слабого человека, питавшего бесконечную любовь к Курагину, — сидели в первой комнате за чаем.
В большом кабинете Дологова, снятом со стен до потолка персидскими коврами, медвежьими шкурами и оружием, Поющего Солокова в Дороге Бешмете и сапогах перед открывшимся бюро, на котором лежали десятки и пачки денег. Анатолий в расстегнутой форме прошел из комнаты, где сидели свидетели, через кабинет в подсобку, где его лакей, француз с другими сложили последние вещи.Шелахов посчитал деньги и записал.
«Ну, — сказал он, — за хвост надо отдать две тысячи».
«Ну давай, дай мне», — сказал Анатолий.
«Макарка (их так звали Макарин), это бескорыстно тебе в огонь и в воду . Ну, партитура закончена, — сказал Солохов, показывая ему записку. — Итак?
«Да, конечно», — сказал Анатолий, по-видимому, не слушая Дологова и с улыбкой, которая не сходила с его лица, с нетерпением ожидая встречи.

О параллельном и секущем.

Вне русскоязычной литературы о теореме Фалеса иногда ссылаются на другую теорему о планиметре, а именно на утверждение, что вписанный угол, основанный на диаметре окружности, является прямым. Открытие этой теоремы действительно приписывается Фалесу, что является свидетельством лагеря.

Формулировка

Если один из двух непосредственно откладывать последовательно несколько равные отрезки и через их концы проводить параллельную прямую, пересекающую вторую прямую, то они будут отрезаны ко вторым прямым равным отрезкам.

Более общая формулировка, также называемая теоремой о пропорциональных сегментах

Параллельная прямая линия, отрезанная на последовательных пропорциональных сегментах:

A 1 A 2 B 1 B 2 = A 2 A 3 B 2 B 3 = A 1 A 3 B 1 B 3. (\ DisplayStyle (\ FRAC (A_ (1) A_ (2)) (b_ ( 1) b_ (2))) = (\ FRAC (A_ (2) A_ (3)) (B_ (2) B_ (3))) = (\ FRAC (A_ (1) A_ (3) ) (b_ (1) b_ (3))).)

Примечания

  • Теорема не имеет ограничений на взаимное расположение секущей (верно как для пересекающихся прямых, так и для параллельных).Также не имеет значения, где находятся отрезки на секущей.
  • Теорема Фалеса является частным случаем теоремы о пропорциональных отрезках, поскольку равные отрезки могут считаться пропорциональными отрезками с коэффициентом пропорциональности 1.

Доказательство в случае последовательного

Рассмотрим вариант с несвязанными парами отрезков: пусть угол пересекает прямую A A 1 | | B B 1 | | C C 1 | | D d 1 (\ displaystyle aa_ (1) || bb_ (1) || cc_ (1) || dd_ (1)), где A B = C D (\ DisplayStyle AB = CD).

  1. Прорежьте точки A (\ DisplayStyle A) и C (\ DisplayStyle C) прямо, параллельно другой стороне угла. A B 2 B 1 A 1 (\\ DISPLAYSTYLE AB_ (2) B_ (1) A_ (1)) и C D 2 D 1 C 1 (\\ DisplayStyle CD_ (2) D_ (1) C_ (1)). По свойству Pollogram: AB 2 = A 1 B 1 (\ DisplayStyle AB_ (2) = A_ (1) B_ (1)) и C d 2 = C 1 D 1 (\ displaystyle CD_ (2 ) = С_ (1) d_ (1)).
  2. Треугольники △ A B B 2 (\\ DisplayStyle \\ BigtrianGeUp ABB_ (2)) и △ C d d 2 (\\ DisplayStyle \\ BigTRIANGEUP CDD_ (2)) равны на основании второго признака равенства треугольников

Доказательство в случае параллельного прямого

Проведем прямо до н.э.. Уголки ABC, и BCD. равны внутренним шкафам, лежащим в основе параллельных прямых AB, и CD, и Sale BC. , и уголки ACB. и CBD. равно внутренним шкафам, лежащим в основе параллельными прямыми AC и BD. и продажа до н.э. . Затем по второму знаку равенства треугольников треугольники ABC, и DCB. равных. Отсюда следует, что AC = BD. и AB = CD . ■

Варианты и обобщения

Обратная теорема

Если в теореме Фалеса равные отрезки начинаются от вершины (часто в школьной литературе используется такая формулировка), обратная теорема также будет верна. Для пересекающихся секверов он формулируется как:

Таким образом (см. Рис.) Из чего CB 1 Ca 1 = B 1 B 2 A 1 A 2 = … (\ displayStyle (\ FRAC (CB_ (1)) (Ca_ (1))) \ u003d (\ FRAC (B_ (1) B_ (2)) (A_ (1) A_ (2))) = \ ldots), следует, что A 1 B 1 | | A 2 B 2 | | … (\\ displaystyle a_ (1) b_ (1) || a_ (2) b_ (2) || \\ ldots).

Если последовательность параллельна, то необходимо потребовать равенства отрезков на обеих секантиях между собой, иначе это утверждение станет некорректным (контрпример — трапеция, пересекаемая линией, проходящей через середину оснований).

Эта теорема используется в навигации: столкновение судов, движущихся с постоянной скоростью, неизбежно, если сохраняется направление от одного судна к другому.

Лемма Соллертинский

Следующее утверждение, двойственное лемме Соллертина:

Пусть будет F (\ DisplayStyle F) — проективное соответствие между точками прямого L (\ DisplayStyle L) и прямого M (\ DisplayStyle M). Тогда много прямых X F (X) (\ DisplayStyle XF (X)) будет много касательных к каким-то

ShT-2020-03.indd

% PDF-1.6
%
739 0 объект
>
эндобдж
736 0 объект
> поток
Adobe InDesign CS3 (5.0) 2020-07-09T16: 17: 33 + 03: 002020-07-09T16: 10: 08 + 03: 002020-07-09T16: 17: 33 + 03: 00application / pdf

  • ShT-2020-03.indd
  • Пользователь
  • Acrobat Distiller 8.1.0 (Windows) uuid: ae6fd2ab-c4d9-4b07-8f21-5b3c7f503511uuid: 5523ecd4-6370-46c9-b0f5-a63bd87df766

    конечный поток
    эндобдж
    796 0 объект
    > / Кодировка >>>>>
    эндобдж
    719 0 объект
    >
    эндобдж
    720 0 объект
    >
    эндобдж
    791 0 объект
    >
    эндобдж
    731 0 объект
    >
    эндобдж
    732 0 объект
    >
    эндобдж
    733 0 объект
    >
    эндобдж
    734 0 объект
    >
    эндобдж
    735 0 объект
    >
    эндобдж
    581 0 объект
    >
    эндобдж
    599 0 объект
    >
    эндобдж
    602 0 объект
    >
    эндобдж
    605 0 объект
    >
    эндобдж
    608 0 объект
    >
    эндобдж
    611 0 объект
    >
    эндобдж
    614 0 объект
    >
    эндобдж
    617 0 объект
    >
    эндобдж
    620 0 объект
    >
    эндобдж
    623 0 объект
    >
    эндобдж
    625 0 объект
    > поток
    h ޜ Zq «= P> LYrĮeVI & ȳ3i RL \ | Iw 랙 Y

    Общие свойства треугольника и параллелограмма.Диагональный параллелограмм недвижимости

    Муниципальное бюджетное образовательное учреждение

    Савинская общеобразовательная школа

    Научно-исследовательская

    Параллелограмм и его новые свойства

    Выполнила: ученица 8Б класса

    МБОУ Савинская Сош

    Кузнецова Светлана, 14 лет

    Руководитель: Учитель математики

    Тульчевская Н.А.

    стр. Савино

    Ивановская область, Россия

    2016.

    I. Введение ________________________________________________________________________

    II. Из истории параллелограмма ___________________________________ стр. 4

    III Дополнительные свойства параллелограмма ______________________ стр. 4

    IV. Подтверждение свойств _____________________________________ стр. 5

    V. Решение задач с использованием дополнительных свойств __________ стр. 8

    Vi.Применение свойств параллелограмма в жизни ___________________ стр. 11

    VII. Заключение _____________________________________________ стр. 12

    VIII. Литература _____________________________________________ стр. 13

      Введение

    « Среди равных умов

    для эквити других условий

    выше того, кто знает геометрию»

    (Blaze Pascal).

    При изучении темы «Параллелограммы» на уроках геометрии мы рассматривали два свойства параллелограмма и три знака, но когда начали решать задачи, оказалось, что этого недостаточно.

    У меня вопрос, а есть ли у параллелограмма еще и свойства, и как они помогут при решении задач.

    И я решил изучить дополнительные свойства параллелограмма и показать, как их можно применить для решения задач.

    Предмет исследования
    :
    параллелограмм

    Объект исследования
    : Свойства поллограммы
    Цель работы:

      формулировка и доказательство дополнительных свойств параллелограмма, которые не изучаются в школе;

      использование этих свойств для решения проблем.

    Задач:

      Найти дополнительную литературу по изучаемой теме;

      Изучите дополнительные свойства параллелограмма и докажите их;

      Покажите использование этих свойств для решения проблем;

      Рассмотрим использование свойств параллелограмма в жизни.
      Методы исследования:

      Работа с учебной и научно-популярной литературой, ресурсами сети Интернет;

      Изучение теоретического материала;

      Выбор круга задач, которые можно решить с помощью дополнительных свойств параллелограмма;

      Наблюдение, сравнение, анализ, аналогия.

    Продолжительность исследования
    : 3 месяца: январь-март 2016 г.

      1. Из истории параллелограмма

    В учебнике геометрии мы читаем следующее определение параллелограмма: Поллограмм — такой четырехугольник, у которого противоположные стороны попарно параллельны

    Слово «параллелограмм» переводится как «параллельные прямые» (от греческих слов Parallelos — параллель и грамм — линия), этот термин ввел Евклид.В своей книге «Начинающий» Евклид доказал следующие свойства параллелограмма: противоположные стороны и углы параллелограмма равны, а диагональ делит его пополам.
    О точке пересечения параллелограмм Евклида не упоминается. Только к концу средневековья была разработана полная теория параллелограммов и только в XVII веке в учебниках появились теоремы о параллелограммах, которые доказываются с помощью теоремы Евклида о свойствах параллелограмма.

    III

    Дополнительные свойства параллелограмма

    В учебнике по геометрии приведены всего 2 свойства параллелограмма:

      Противоположные углы и стороны равны

      Диагональ параллелограмма пересекается, а точка пересечения делится пополам

    В различных источниках геометрии можно найти следующие дополнительные свойства:

      Количество смежных углов Поллограмм, равный 180 0

      Биссектриса угла параллелограмма отсекает от него равносторонний треугольник;

      Биссектрисы противоположных углов параллелограмма лежат на параллельных прямых;

      Биссектриса смежных углов Параллелограмм пересекаются под прямыми углами;

      Биссектриса всех углов параллелограмма с пересечением образует прямоугольник;

      Расстояния от противоположных углов параллелограмма до одной и той же диагонали равны.

      Если в параллелограмме соединить противоположные вершины с серединой противоположных сторон, то получится другой параллелограмм.

      Сумма квадратов диагоналей параллелограмма равна двойной сумме квадратов его смежных сторон.

      Если в параллелограмме двух противоположных углов провести высоты, то получится прямоугольник.

    IV.

    Доказательство свойств параллелограмма

      Количество смежных углов Параллелограмм равен 180
      0

    Dano :

    ABCD — параллелограмм

    Prove

    A +.
    Б =.

    Доказательства:

    AI
    B-изогнутые односторонние углы с параллельным прямым солнцем AD и Sale AV, то есть
    A +.
    Б =.

    2

    Дано: ABCD
    параллелограмм,

    АК-Биссектрис
    И.

    Доказательство AVK — равно

    Доказательства:

    1)
    1 =
    3 (перекрестно проходит под солнцем AD и Sale AK)

    2)
    2 =
    3 т.К. Ак — бисектриса,

    Так 1 =
    2.

    3) АВК — нерешенный т. 2. 2 угла треугольника равны

    . Биссектрисный угловой параллелограмм образует равносторонний треугольник

    3

    Дано: AVD — параллелограмм,

    Ak — Bissektris A,

    CP — Bissektris C.

    Prove AK ║ ср.

    Обоснование:

    1) 1 = 2 т.К. Ак-bissectrice

    2) 4 = 5 т.к. ср — Bissektris

    3) 3 = 1 (Малые углы

    Sun ║ AD и AK-SECOND),

    4 ) A = C (по свойству параллелограмма), значит 2 = 3 = 4 = 5.

    4) из пунктов 3 и 4 следует, что 1 = 4, и этим углам соответствуют для прямого АК и КП и Южного Солнца,

    Итак, АК ║ КП (на основе параллельности прямого)

    .Биссектриса противоположных углов Параллелограмм лежит на параллельных прямых

      Биссектриса смежных углов Параллелограмм пересекается под прямым углом

    Дано: AVD — параллелограмм,

    Ak-bisectaris A,

    DR Bissectrix D

    Prove DR AK.

    Свидетельство:

    1) 1 = 2, потому что AK — Bissektris

    Пусть, 1 = 2 = x, тогда a = 2x,

    2) 3 = 4, потому что DR — Bissectrice

    Пусть, 3 = 4 = y, тогда d = 2y

    3) a + d = 180 0, т.к. сумма смежных углов Параллелограмма равна 180

    2) Рассмотрим A OD.

    1 + 3 = 90 0, тогда

    5. Биссектрика всех углов Параллелограмм с пересечением образуют прямоугольник

    Дано: AVD — параллелограмм, ak-bisectaris A,

    DR, биссектриса D,

    Cm -Bistektris C,

    BF -Bissectrice b.

    Доказательство : КРНС -ПРЯУГОН

    Доказательство:

    Исходя из предыдущего свойства 8 = 7 = 6 = 5 = 90 0,

    так что КРНС — яркий.

      Расстояния от противоположных углов параллелограмма до одной и той же диагонали равны.

    Дано: ABCD-параллелограмм, as-диагональ.

    VC Ac DP. AC

    Prove BC = DR

    Evidence: 1) DCR = KAB, как внутренние замыкания, лежащие в основе AB ║ CD и SECOND AC.

    2) АКБ =. CDR (сбоку и два регулировочных угла AV = CD Cd p = AB K).

    А в равных треугольниках соответствующие стороны равны, значит DR = BC.

      Если в параллелограмме соединить противоположные вершины с серединой противоположных сторон, то получится другой параллелограмм.

    Дано: ABCD-параллелограмм.

    Докаж ВКДР — параллелограмм.

    Свидетельство:

    1) BR = KD (ad = bc, указывает на и r

    делит эти партии пополам)

    2) BP ║ KD (лежит на AD BC)

    Если в противоположные стороны четырехугольника равны и параллельны, значит, этот четырехугольник -паралограмма.

      Если в параллелограмме двух противоположных углов провести высоты, то получится прямоугольник.

      Сумма квадратов диагоналей параллелограмма равна двойной сумме квадратов его смежных сторон.

    Дано: ABCD — параллелограмм. BD и AC — диагональ.

    Prove AC 2
    + тд. 2
    = 2 (AB 2
    + AD 2
    )

    Доказательства: 1) СПРОСИТЬ:


    AC
    ² =


    +

    2) Б.
    R D.
    :
    BD.
    2
    =
    Б.
    R 2
    + Р. Д.
    2
    (по теореме Пифагора)

    3 )
    AC
    ² +
    BD.
    ² = SCQ + A.
    К² +. Б.
    R² + R. D.
    ²

    4) СК = БП = Н (высота )

    5) AC. 2
    + Б. Д.
    2
    =
    H.
    2
    +
    А.
    К 2
    +
    H.
    2
    + Р. Д.
    2

    6)
    Пусть будет
    Д.
    К =. А.
    P = H. , затем C.
    К Д.
    :
    H.
    2
    =
    CD
    2
    — Н. 2

    согласно теореме Пифагора )

    7) как xt + в D.
    ² = S. D.
    2
    — x² + ак 1
    ² +
    CD
    2
    -H. 2
    + Р. Д.
    2
    , г.

    As² + B. Д.
    ² = 2s D.
    2
    -2x 2
    +
    А.
    К 2
    + Р. Д.
    2

    8) А.
    К = AD +.
    ч. , г.

    R D = ad
    ч. , г.

    As² + B. D.
    ² = 2. CD
    2
    -2x 2
    + ( н.э.
    + х) 2
    + ( н.э.
    ) 2
    , г.

    AC ² +

    IN D² = 2.
    ИЗ D²-2
    ч. ² + AD.
    2

    + 2AD
    ч. +

    ч. 2

    + нашей эры
    2

    -2ад.
    ч. +

    ч. 2

    ,

    AC ² +

    IN D² = 2cd.
    2

    + 2AD
    2

    = 2 (CD
    2

    + нашей эры
    2

    ).

    В.

    . Решение задач с использованием этих свойств

      Точка пересечения биссектрисы двух углов параллелограмма, примыкающих к одной стороне, принадлежит противоположной стороне. Меньшая сторона параллелограмма равна 5.
      . Найдите это с самой стороны.

    Дано: ABCD — параллелограмм,

    AK — Bissektris
    AND,

    D K — биссектриса
    D, av = 5

    Найти : Sun.

    беспорядок

    Решение

    Поскольку AK — Bissektris
    A, то AVC является равным.

    Потому что DK — биссектриса
    D, T. DCK — Wireless

    DC = CK = 5

    Тогда Sun = VK + SC = 5 + 5 = 10

    Ответ: 10.

    2. Найдите периметр параллелограмма, если биссектриса одного из его углов делит сторону параллелограмма на отрезки 7 см и 14 см.

    1 корпус

    Дано:
    AND,

    VK = 14 см, kc = 7 см

    Найти: Параллелограмм

    9000d Sun VK + kc = 14 + 7 = 21 (см)

    Поскольку AK — Bissektris
    A, то AVC является равным.

    AV = VK = 14 см

    Тогда p = 2 (14 + 21) = 70 (см)

    происходящее

    Дано: ABCD — параллелограмм,

    DK — биссектриса
    D,

    ВК = 14 см, kc = 7 см

    Найти : P параллелограмм

    Решение

    Солнце = ВК + kc = 14 + 7 = 21 (см)

    Потому что ДК — биссектриса
    D, T.DCK — Wireless

    DC = CK = 7

    Тогда p = 2 (21 + 7) = 56 (см)

    Ответ: 70см или 56 см

    3. Детекторами параллелограмма являются равны 10 см и 3 см. Биссектриса двух углов, примыкающих к большей стороне, делит противоположную сторону на три сегмента. Найдите эти сегменты.

    1 случай: биссектрисы пересекаются вне параллелограмма

    Дано: ABCD — параллелограмм, ак — биссектриса
    AND,

    DK — биссектриса
    D, av = 3 см, солнце = 10 см

    Найти : VM, Mn, NC

    Решение

    Поскольку AM — Bissektris
    A, то AVM является равным.

    Т.к. DN — Bissektris
    D, T. DCN — равно

    DC = CN = 3

    Тогда Mn = 10 — (Bm + NC) = 10 — (3 + 3) = 4 см.

    2 случай: бисектрис пересекаются внутри параллелограмма

    Поскольку An — Bissektris
    A, то Avn является равновесием.

    АВ = Б. Н.
    = 3
    D.

    А раздвижная сетка — для перемещения на необходимое расстояние в дверном проеме

    Механизм параллелограмм — Механизм четырехжильный, звенья которого представляют собой параллелограммы.Применяется для реализации поступательного движения с помощью навесных механизмов.

    Параллелограмм с неподвижным звеном — Одно звено неподвижно, противоположное совершает колебательное движение, оставаясь параллельным неподвижным. Два соединенных между собой параллелограмма дают конечному звену две степени свободы, оставляя его параллельным неподвижному.

    Примеры: автобусы, погрузчики, треноги, подвески, автомобильные подвески.

    Параллелограмм с фиксированным шарниром — свойство параллелограмма используется для поддержания постоянного отношения расстояний между тремя точками.Пример: Исходный пантограф — устройство для масштабирования чертежей.

    Ромб — Все звенья одинаковой длины приближаются (затягиваются) Пара противоположных шарниров приводит к раскрытию двух других петель. Все ссылки работают на сжатие.

    Примеры — автомобильный ромбовидный домкрат, трамвайный пантограф.

    Ножницы или Х-образный механизм , также известный как Нюрнбергские ножницы — вариант рома — два звена, соединенные в середине петли. Достоинства механизма — компактность и простота, недостаток — наличие двух пар скольжения.Два (или более) таких механизма, соединенных последовательно, образуют середину ромба (ов). Используется в слушателях, детских игрушках.

    VII

    Заключение

    Кто с детства занимается математикой,

    развивает внимание, тренирует свой мозг,

    свою волю, повышает настойчивость

    и настойчивость в достижении цели

    Маркус А.

      В процессе работы я доказал дополнительные свойства параллелограмма.

      Я убедился, что применяя эти свойства, вы сможете быстрее решать поставленные задачи.

      Я показал, как эти свойства применяются на примерах решения конкретных задач.

      Я узнал много нового о параллелограмме, чего нет в нашем учебнике по геометрии

      Я убедился, что знание геометрии очень важно в жизни на примерах применения свойств параллелограмма .

    Цель моей исследовательской работы выполнена.

    Насколько важны математические знания, факт, что награда учреждена тому, кто издаст книгу о человеке, который жил без помощи математики. Эту премию еще не получил ни один человек.

    VIII.

    Литература

      1. Погорелова А.А. Геометрия 7-9: Учебник для общего образования. Учреждения-М.: Просвещение, 2014

        Л.С. Танасян и др. Геометрия. Дополнительный. К учебнику 8 кл: ученица. Пособие для школьников и классов с изображением. Исследования. Математика. — М .: Вита Пресс, 2003

        Интернет-ресурсы

        Материалы википедии

    Параллелограмм — это четырехугольник, у которого параллельны противоположные стороны. На следующем рисунке показаны параллелограммы ABCD. У него сторона AB параллельна стороне CD, а сторона BC параллельна стороне AD.

    Как вы уже догадались, параллелограмм представляет собой выпуклый четырехугольник. Рассмотрим основные свойства параллелограмма.

    Свойства параллелограмма

    1. В параллелограмме противоположные углы и противоположные стороны равны. Докажем это свойство — рассмотрим параллелограмм, представленный на следующем рисунке.

    Диагональ

    BD делит его на два равных треугольника: ABD и CBD. Они равны со стороны BD и двух регулировочных углов, поскольку углы крестовин под блоком BD параллельны прямым BC и AD и AB и CD соответственно.Следовательно, AB = CD и
    BC = AD. А из равенства углов 1, 2, 3 и 4 следует, что угол А = угол 1 + угол 3 = угол 2 + угол 4 = угол С.

    2. Диагональ параллелограмма точки пересечения делится пополам. Пусть точка o является точкой пересечения диагоналей параллелограмма ABCD и AC и BD.

    Тогда треугольник AOB и треугольник COD равны друг другу на стороне и двух прилегающих к нему углах. (Ab = Cd Так как это противоположные стороны параллелограмма.А угол 1 = угол 2 и угол 3 = угол 4 как увеличение нижележащих углов при пересечении прямых AB и CD с помощью AC и BD соответственно.) Из этого следует, что AO = OC и OB = OD, что AO = OC и OB = OD что и надо было доказать.

    Все основные свойства показаны на следующих трех фотографиях.

    На указателе Parale-le-Lo Gram-ma

    1. Определение и основные свойства параллелограмма

    Nach, с тем, что UPR-de-le-le-Lo gram-Ma запомнит.

    OPRA-de-les. Параллелограмм — прозвище Che-you-ka-уголь, на co-ro, каждые два про-типа-в-слое-ложных роликах (см. Рис. Один).

    Рис. 1. PA-RAL-LE-Lo Gram

    Erees oS-новый PA-RAL-LE-Lo Gram-Ma :

    Для того, чтобы иметь ВОЗ Поль-Зо-Валент, все эти потребности-ди-ди-бэ уве-рен, Ф-ГУ-РА, О «Мы говорим о» Па-Рал-ле -Меньше грамма. Для этого необходимо знать такие факты, как пример Параль-ле-Ло Грам-Ма.Первые два из них мы se-year-nya и Raspie Rome.

    2. Первая особенность параллелограмма

    Теорема. Перья Parale-le-Lo Gram-Ma. Если есть два pro-second-false-nic-ours во втором-false-ours are equal и Parale-Lel-us, то это che-you-fl-уголь-прозвище параллелограмм . .

    Рис. 2. Ручка с Para Ral-le-Lo Gram-ma

    Доказательства. Пров-ве-дем в Че-ты-Ри-Нал-Нал (см. Рис. 2), она когда-то-Би-ла на своих двух тре-уголь-ни-ка.В Пи-Шеме, что мы знаем об этих тре-уголь-никах:

    по пер-ин-му, на ряд-ку-ве-ве-пробе.

    Из Ра-Вен-Ну-в-глаз Тре-Уголь-Ни-Бухта, которая является причиной-ку-Раал-Лел-но-си, прямолинейный Че — их семя. У нас это:

    Du-ka-it.

    3. Параллелограмм второго знака

    Теорема. ВТО-рой со знаком Parale Le-Lo Gram-Ma. Если каждый из двух угольных нике, каждый из двух про-Ti-in-up-пластов равны, тогда этот параллелограмм che-you-fl-уголь-ник ..

    Рис. 3. ВОЗ-ВОЗ PA-RAL-LE-LO-MA

    Доказательства. Проведем в Че-Ма-Нал-На-Нале (см. Рис. 3), это когда-то-би-ва-он на два тре-уголь-ни-ка. В Пи-Шем, что мы знаем об этих Тре-Уголь-Ни-ках, ИС-Хо-Диа из формы-Му-Ли-Ки Тео-ре-мы:

    Согласно Тре-Му, ат-Ку-Ку Ра-Веренов-Тре-Уголь-Ни-Бухта.

    Из Ra-Ven-Spenda Tre-Coal-Ni-noves, а также по причине-ku-Raal-Lel-no-si право с ne-re-che Sea-ku In Lou-Chul:

    PA-RAL-LE-LOO-грамм в соответствии с защитой de leu.Q.E.D.

    Du-ka-it.

    4. Пример использования первого признака параллелограмма

    Ras-проверено в действии на трассе, отличной от NE-Ski-Raal-Le-Lo Gram-Ma.

    Например 1. В тебе-пучке че-ты-дех-уголь-ни-ке находим: а) Уголки Че-ты-дех-уголь-ни-ка; б) сто-по-колодец.

    Решение. Исоб-ра и рис. четыре.

    PA-RAL-LE-LO-GRAM PER-MU-MU-KU-KU PA-RAL-LE-LOO-MA MA.

    И. Согласно Параль-ле-Ло Грам-Ма, о про-Ти-По-Ло-Ло грамм-Ма, по сумме угловых, Леви до ста рун.

    B. Согласно работе Ра-Вен-Прос-Ти-Флайт Стро-Рон.

    re-Tiy с Para Ral-le-Lo Gram Ma

    5. Повторение: определение и свойства Парлиаллограмма

    On-Pom-him that параллелограмм — Это то, что вы-ро-уголь-прозвище, в one-ro-th-Ti-Par-Paradh-Para-Raal-Lel-we. То есть, если — грамм Параль-ле-Ло, то (см. Рис. 1).

    Parale-le-Lo-Lo-La-La-дает множество свойств: Pro-Ti-B-Fallest углы равны (), Pro-Ti-in-Flap-Po — равны ().Кроме того, dia-go-on-La Parale-le-Lo Gram Ma в точке PE-CE-Lama, сумма углов в Paral-le-Lo gram-Ma, равная любой ста -рублей, равно

    Но для того, чтобы Поль-Зоопарк во всех этих, необходимость в необходимости быть аб-со-лют-но уве-рен-сегодня состоит в том, что Рас-Юг Ри-Ва-Эх Че-ты- roh-уголь-ник — Parale-le-Lo Gram. Для этого и поддержания под знаком Parale-le-Lo Gram-Ma: то есть фактов, из которых можно сделать однозначный вывод.что люди Яв-Ла-Лия-Сиа Параль-ле-Ло Грам-Мам. В предыдущем уроке нас уже воспитали двое. Этот час — Римский Трий.

    6. Третий знак параллелограмма и его доказательство

    Если в том-вы-рых-уголь-ни-ке ди-он-он в точке PE-SE-ламы, то Дэн-ни Рих-уголь-прозвище Ява-Лаа-Сиа Параль-ле-Ло Грам -Мама.

    Дано:

    Прозвище Che-you-roh-уголь; ; .

    До качества:

    Параллелограмм.

    Обоснование:

    Чтобы усомниться в данном факте, необязательно-дитя-ди-ми-дю-ро-лель-лель грамм-ма Парал-лель-Ло-Ма. И Паралель-Леллеан чаще всего идет до Ка-Зы-Вася, через Рен Вен-Рен-Рен их на пересечении лесных углов с этими прямыми. Таким образом, он-луч, он-ши-ва-смта, до-й-ши-вока, Ду-Ка-Прос-Тел-Тель-Гуа Парале Ла-Ло Грам Ма: через Рета-Тре-Уголь-Ни -Cove.

    Ду-Ка-Жа Рас-Вен-эти Tre-Coal Ni-Cove. Дей-Тел-тел-а, из Слой-Вия после:.Кроме того, есть углы углов — вернее, они равны. Т.е.:


    ( per-re-игла tre-уголь — для двести-ро и уголка между ними).

    Из Tre-Coal-Ni-ni-cork: (поскольку пересечение равняется углам на рении с этими прямыми и se-ку). Кроме того, Tre-уголь-Ni-ni-knit-роман, который. Знай-чит, мы в Лу-чи, что в Тысячи-уголь-ни-ке двести стержней равны и Паралель-Лель. Согласно пер-ин-Му, ат-ку-ка-Раль-ле-Ло-Ма: — Параль-ле-Ло Грам.

    Ду-ка-ит.

    7. Пример задания на третий признак параллелограмма и обобщение

    Ras-проверено в действии на не-NE-RU-RA-le-Lo Gram-Ma.

    В действии 1

    Дано:


    параллелограмм; . — Ce-re-di-on, — se-re-di-on, — se-re-di-on, — se-re-di-on, — se-re-di (см. Рис. 2).

    До качества: — Параль-ле-Ло грамм.

    Обоснование:

    Ноу-чит, в точке Тхие-Уголь-Ни-Ке Диа-На-Да-Лят-Сми-Лима.В Tre-T-Mu, at-ku-ka-Raal-le-Lo gram-Ma, отсюда презренно, что — Parale-le-Lo Gram.

    Ду-ка-ит.

    Если про-заказ Ana Liz T-Ra-le-Lo Gram-Ma, то вы можете его заработать, что этот кейс является признаком совместного ветеринара. У него есть свой Parale-le-Lo. Грамм-ма. То есть тот факт, что диа-го-он-ли-лят-ся слабый, Яв-ла — это не про-сто Параль-ле-Ло грам-ма, И его от-Ли-Чи -тел, ха-рак-те-река-че-как, на которую он может де выливаться из глотки Че-ты-ро-уголь-ни-ков.

    ИСТОЧНИК

    http://interneturok.ru/ru/school/geometry/8-klass/chyotyrehugolniki/priznaki-parallelogramma.

    http://interneturok.ru/ru/school/geometry/8-klass/chyotyrehugolniki/tretiy-priznak-parallelogramma.

    http://www.uchportfolio.ru/users_content/675f9820626f5bc0afb47b57890b466e/images/46tthxq8j4y.jpg

    http://cs10002.vk.me/u311

    /116260458/x_56d40dd3.jpg

    http://wwww.tepka.ru/geometriya/16.1.gif.

    Тематический урок

    • Диагональный параллелограмм объекта.

    Задачи урока

    • Познакомьтесь с новыми определениями и вспомните некоторые из уже изученных.
    • Сформулируйте и докажите свойство диагоналей параллелограмма.
    • Научитесь применять свойства фигур при решении задач.
    • Развивающие — развивают внимание учащихся, усидчивость, усидчивость, логическое мышление, математическую речь.
    • Образовательные — через урок воспитывать внимательное отношение друг к другу, прививать умение слушать товарищей, взаимное исполнение, самостоятельность.

    Задачи урока

    • Проверить умение учащихся решать задачи.

    План урока

    1. Вступительное слово.
    2. Повторение ранее изученного материала.
    3. Поллограмма, ее свойства и признаки.
    4. Примеры задач.
    5. Независимая проверка.

    Введение

    «Крупное научное открытие дает решение большой проблемы, но и в решении любой задачи есть зерно открытия.»

    Свойство противоположных сторон параллелограмма

    У параллелограмма противоположные стороны равны.

    Доказательства.

    Пусть ABCD будет этим параллелограммом. И пусть его диагонали пересекаются в точке О.
    Так как Δ AOB = Δ COD по первому основанию равенства треугольников (∠ aob = ∠ COD, как и вертикаль, AO = OC, DO = OB, по свойству диагональ параллелограмма), то AB = CD.Аналогично из равенства треугольников Вос и ДОА следует, что ВС = DA. Теорема доказана.

    Свойство параллелограмма противоположных углов

    У параллелограмма противоположные углы равны.

    Доказательства.

    Пусть ABCD будет этим параллелограммом. И пусть его диагонали пересекаются в точке О.
    Из доказанного в теореме о свойствах противоположных сторон параллелограмм Δ ABC = δ CDA в трех сторонах (AB = CD, BC = DA из доказанного, AC равен общий).Из равенства треугольников следует, что ∠ ABC = ∠ CDA.
    Также доказано, что ∠ dab = ∠ BCD, что следует из ∠ ABD = ∠ CDB. Теорема доказана.

    Диагональный параллелограмм недвижимости

    Диагональ параллелограмма пересекается, а точка пересечения делится пополам.

    Доказательства.

    Пусть ABCD будет этим параллелограммом. Проводим диагональ переменного тока. Отметим на нем середину O. На продолжение отрезка DO отложите отрезок OB 1, равный DO.
    Согласно предыдущей теореме AB 1 CD — параллелограмм. Следовательно, направьте AB 1 параллельно DC. Но через точку A можно провести только один прямой параллельный DC. Итак, прямая AB 1 совпадает с прямой AB.
    Это также доказывает, что BC 1 совпадает с BC. Итак, точка C совпадает с 1. Параллелограмм ABCD совпадает с параллелограммом AB 1 CD. Следовательно, диагональ параллелограмма пересекается, а точка пересечения делится пополам. Теорема доказана.

    В учебниках для обычных школ (например, в Погорелове) доказано так: диагонали параллелограммов разделены на 4 треугольника.Рассмотрим одну пару и выясним — они равны: основания у них противоположные стороны, примыкающие к ней соответствующие углы равны вертикали с параллельными прямыми. То есть отрезки диагоналей попарно равны. Все.

    Это все?
    Выше доказано, что точка пересечения делит диагональ пополам — если есть. Само существование приведенных выше рассуждений никоим образом не доказывает. То есть часть теоремы «диагональный параллелограмм пересекается» остается недоказанной.

    Забавно, что доказать эту часть намного сложнее. Это, кстати, следует из более общего результата: любая выпуклая четырехконтактная диагональ будет пересчитана, никаких неприятных не будет.

    О равенстве треугольников по бокам и двух поправках к нему (второй признак равенства треугольников) и др.

    Теорема о равенстве двух треугольников на стороне и двух смежных углов Фалеса нашла важное практическое применение.В гавани Милеты построили дальномер, определяющий расстояние от корабля до моря. Он представлял собой три сбитые сваи A, B и C (AB = Sun) и место прямого SC, перпендикулярного. Когда корабль появляется на прямом SC, была точка D такая, что точки d ,. In и e оказались на одной прямой. Как видно из рисунка, расстояние от КД на земле — это желаемое расстояние до корабля.

    Вопросы

    1. Диагональ квадрата точки пересечения делится пополам?
    2. Диагональный параллелограмм равен?
    3. Управляющие углы параллелограмма равны?
    4. Слово определение параллелограмма?
    5. Сколько знаков у параллелограмма?
    6. Может ли ромб быть параллелограммом?

    Список использованных источников

    1. Кузнецов А.В., учитель математики (5-9 классы), Киев
    2. «ЕГЭ 2006. Математика. Учебные материалы для обучения студентов / Рособрнадзор, Исоп — М .: Интеллект-Центр, 2006»
    3. Мазур К.И. «Решение основных соревновательных задач по математике сборника под ред. М.И. Сканави»
    4. Атанасян Л.С., Бутузов В.Ф., Кадомцев С.Б., Позняк Е.Г., Юдина И.И. «Геометрия, 7 — 9: Учебник для общеобразовательных учреждений»

    Над уроком работали

    Кузнецов А.V.

    Purknak S.A.

    Евгений Петров

    Задать вопрос о современном образовании, выразить идею или решить уреранную проблему можно. Образовательный форум Где на международном уровне собирается образовательный совет свежих мыслей и действий. Создание blog Вы не только повысите свой статус грамотного учителя, но и внесете весомый вклад в развитие школы будущего. Гильдия лидеров образования Открывает двери для высококлассных специалистов и приглашает к сотрудничеству в направлении создания лучших школ мира.

    Предмет> Математика> Математика 8 класс

    При решении задач по данной теме кроме основных свойств параллелограмм и соответствующие формулы можно запомнить и применить следующим образом:

    1. Биссектриса внутреннего угла Параллелограмм разрезает от него равносторонний треугольник
    2. Биссектрисы внутренних углов, прилегающие к одному боковому параллелограмму, взаимно перпендикулярны
    3. Биссектрисы, выходящие из противоположных внутренних углов, параллелограмм, параллельные друг другу или лежащие на одной прямой
    4. Сумма квадратов диагоналей параллелограмма равна сумме квадратов его сторон
    5. Площадь параллелограмма равна половине работы диагоналей на синусоидальном углу между ними

    Рассмотрим задачи при решении этих свойств.

    Задача 1.

    Биссектриса угла с параллелограммом AVD пересекает сторону AD в точке M и продолжение стороны AV на точку A в точке E. Найти периметр параллелограмма, если Ae = 4 , DM = 3.

    Решение.

    1. Абажур с треугольником — стульчик. (Свойство 1). Следовательно, Cd = Md = 3 см.

    2. Треугольник EAM предшествует.
    Следовательно, АЕ = АМ = 4 см.

    3. AD = AM + MD = 7 см.

    4. Периметр АБСД = 20 см.

    Ответ. 20 см.

    Задача 2.

    В выпуклой четырехпусковой диагонали АВД. Известно, что Квадрат треугольников AVD, ACD, ADD равен. Докажите, что эта квадриль — параллелограмм.

    Решение.

    1. Пусть be — высота треугольника AVD, CF — высота треугольника ACD.Так как по условию задания площади треугольников у них тоже общее основание AD, то высота этих треугольников равна. Ve = CF.

    2. ve, CF перпендикулярно AD. Точки входа и выхода расположены с одной стороны относительно прямого AD. Ve = CF. Следовательно, прямое солнце || ОБЪЯВЛЕНИЕ. (*)

    3. Пусть AL — высота треугольника ACD, Bk — высота треугольника BCD. Поскольку по условию задания площади треугольников у них тоже есть общее основание CD, то высота этих треугольников равна.Al = bk.

    4. Al и BK перпендикулярны CD. Точки in и a расположены с одной стороны относительно прямого CD. Al = bk. Следовательно, прямое Av || CD (**)

    5. Из условий (*), (**) потоки — параллелограммы AVD.

    Ответ. Доказано. АВД — параллелограмм.

    Задача 3.

    По бокам самолета и CD параллелограмм ABSD отмечены точками M и H соответственно, так что сегменты VM и HD пересекаются в точке O;

    Решение.

    1. В треугольнике Dom

    2. В прямоугольном треугольнике DNS
    (

    Тогда (Так как в прямоугольном треугольнике катат, лежащий против угла 30 o, равного половине гипотенузы).

    А вот cd = av. Тогда av: nd = 2: 1.

    3.

    4.

    Ответ: AV: HD = 2: 1,

    Задача 4.

    Одна из диагоналей параллелограмма длиной 4√6 имеет угол 60 o, а вторая диагональ имеет такой же угол основания 45 o.Найдите вторую диагональ.

    Решение.

    1. АО = 2√6.

    2. К треугольнику AOD примените теорему о синусах.

    АО / СИН Д = ОД / СИН А.

    2√6 / SIN 45 O = OD / SIN 60 O.

    OD = (2√6Sin 60 O) / SIN 45 O = (2√6 · √3 / 2) / (√2 / 2) = 2√18 / √2 = 6.

    Ответ: 12.

    Задача 5.

    Параллелограмм со сторонами 5√2 и 7√2, меньший угол между диагоналями равен меньшему углу параллелограмма.Найдите сумму длин диагоналей.

    Решение.

    Пусть D 1, D 2 — диагонально параллелограмм, причем угол между диагоналями и меньшим углом параллелограмма равен f.

    1. Отсчитайте
    двух разных путей к его области.

    S abcd = ab · ad · sin a = 5√2 · 7√2 · sin f,

    S abcd = 1/2 AS · cd · sin AOs = 1/2 · d 1 d 2 sin f.

    Получаем равенство 5√2 · 7√2 · sin f = 1 / 2D 1 d 2 sin F или

    2 · 5√2 · 7√2 = d 1 d 2;

    2.Используя соотношение сторон и диагоналей параллелограмма установим равенство

    (AB 2 + AD 2) · 2 = AC 2 + CD 2.

    ((5√2) 2 + (7√2) 2) · 2 = d 1 2 + d 2 2.

    d 1 2 + d 2 2 = 296.

    3. Создайте систему:

    (D 1 2 + d 2 2 = 296,
    (D 1 + d 2 = 140.

    Умножьте второе уравнение системы на 2 и сложите первое.

    Получаем (D 1 + d 2) 2 = 576. Отсюда ID 1 + D 2 i = 24.

    Поскольку d 1, d 2 — длина диагоналей параллелограмма, то D 1 + d 2 = 24.

    Ответ: 24.

    Задание 6.

    Стороны параллелограмма 4 и 6. Острый угол между диагоналями составляет 45 o. Найдите область поллограммы.

    Решение.

    1. Из треугольника AOs, используя теорему косинусов, запишем отношение между стороной параллелограмма и диагоналями.

    АБ 2 = АО 2 + в 2 2 · АО · КОС АО.

    4 2 = (D 1/2) 2 + (d 2/2) 2 — 2 · (D 1/2) · (d 2/2) COS 45 O;

    d 1 2/4 + d 2 2/4 — 2 · (D 1/2) · (D 2/2) √2 / 2 = 16.

    d 1 2 + d 2 2 — d 1 · d 2 √2 = 64.

    2. Аналогичным образом запишите соотношение для треугольника AOD.

    Учитываем то, что

    Получаем уравнение d 1 2 + d 2 2 + d 1 · d 2 √2 = 144.

    3. Имеем систему
    (D 1 2 + d 2 2 — d 1 · d 2 √2 = 64,
    (D 1 2 + d 2 2 + d 1 · d 2 √2 = 144.

    Выживаем из второго уравнения сначала, получаем 2D 1 · d 2 √2 = 80 или

    d 1 · d 2 = 80 / (2√2) = 20√2

    4. S ABCD = 1/2 AS · cd · Sin AOs = 1/2 · d 1 d 2 sin α = 1/2 · 20√2 · √2 / 2 = 10.

    Примечание: В этой и в предыдущей задачах нет необходимости решать полную систему, предполагая, что нам понадобится произведение диагоналей в этой задаче для вычисления площади.

    Ответ: 10.

    Задача 7.

    Площадь параллелограмма равна 96, а его стороны 8 и 15. Найдите квадрат наименьшей диагонали.

    Решение.

    1. S ABCD = AV · AD · SIN Vad.

    Добавить комментарий

    Ваш адрес email не будет опубликован. Обязательные поля помечены *